Параллельное и последовательное подключение резисторов: Онлайн-калькулятор расчета последовательного и параллельного соединения резисторов

Содержание

Параллельное и смешанное соединение резисторов: разные способы подключения сопротивления

Ни одна операция в электронике или электротехнике не обходится без вычисления сопротивления. В этом случае рассматривают только тот участок цепи, в котором находится смешанное соединение резисторов. Инженерам и физикам необходимо понимать то, как именно происходят расчёты в таких схемах. Всего разделяют несколько видов подключения, которые используются в цепях различной сложности.

Последовательное соединение

Выделяют такие способы соединения резисторов: последовательное, параллельное и комбинированное. При последовательном подключении конец первого резистора подключают к началу второго, его часть к третьему. Так действуют со всеми составляющими. То есть все компоненты цепи следуют друг за другом. Через них в таком подключении будет проходить один общий электрический ток. Для таких схем физики применяют формулу, в которой между точками А и В есть только один путь протекания заряженных электронов.

От количества подключённых резисторов зависит сопротивление протекающему электричеству. Чем больше составляющих, тем оно выше.

Его рассчитывают по формуле: R общее = R1+R2+…+Rn, где:

  • R общее — это сумма всех сопротивлений;
  • R1 — первый резистор;
  • R2 — второй компонент;
  • Rn — последняя составляющая в цепи.

Параллельное подключение

Параллельное соединение подразумевает подключение начал резисторов к одной точке, а концов к другой. Сами компоненты при этом расположены на одинаковом расстоянии друг от друга, а их количество не ограничено. По каждой составляющей электричество протекает отдельно, выбирая один из нескольких путей.

Из-за того, что в цепи находится несколько компонентов и путей прохода тока, сопротивление значительно меньше, чем при последовательном соединении. То есть общая сумма противодействия уменьшается пропорционально увеличению количества составляющих. Формула для определения общей суммы противостояния электричеству: 1/R общее = 1/R1+1/R2+…+1/Rn.

В расчётах общее сопротивление всегда должно быть меньше любого из составляющих цепи. Способ вычисления суммы противостояния для схемы из двух резисторов немного отличается: 1/R общее = (R1 х R2)/(R1+R2). Если в системе у компонентов одинаковые показатели сопротивления, то общее число будет равно половине одного из составляющих.

Смешанный вариант

В смешанном соединении сопротивлений комбинируют последовательную и параллельную схему подключений. В этом случае несколько компонентов соединяют одним способом, а другие — вторым, но все они включены в одну цепь. В физике такой метод соединения называют последовательно-параллельным.

Для вычисления суммы противостояния электричеству схему нужно разбить на мелкие участки, в которых резисторы подключены одинаковым способом. Затем расчёты проводят по алгоритму:

  • в цепи с параллельно соединёнными компонентами высчитывают эквивалентное сопротивление;
  • после этого высчитывают противостояние на последовательно подключённых участках схемы;
  • наглядную иллюстрацию нужно перерисовать, обычно получается цепь с последовательным соединением резисторов;
  • рассчитывают сопротивление в новой схеме по одной из двух формул.

Лучше понять методы вычислений поможет пример. Если в схеме всего пять компонентов, они могут располагаться по-разному. Начало первого резистора подключено к точке А, конец — к В. От неё идёт отдельная схема с комбинированным соединением. Вторая и третья составляющие находятся на последовательной линии, четвёртый компонент параллелен им. От конечной точки этой цепи — Г — исходит последний резистор.

Сначала высчитывают сумму сопротивления последовательного участка внутренней схемы: R2+R3. После этого цепь перерисовывают так, чтобы второй и третий компоненты были соединены в один. В результате внутренняя цепь имеет параллельное подключение. Теперь высчитывают её противостояние: (R2,3xR4)/(R2,3+R4). Можно второй раз изобразить полученную цепь.

В схеме будет три резистора, соединённые последовательным методов. Причём средний включает параметры второго, третьего и четвёртого компонента.

Теперь можно узнать общую сумму сопротивлений. Для этого складывают показатели противостояний электричеству первого, пятого и остальных составляющих. Формула будет иметь вид: R1+(R2,3xR4)/(R2,3+R4)+R5. Можно сразу подставить в неё все параметры компонентов.

На практике последовательный и параллельный метод соединения используются редко, ведь в приборах схемы обычно сложные. Поэтому в цепях резисторы часто соединены комбинированным способом. Сопротивление в таких случаях высчитывают пошагово.

Если сразу выводить числа в общую формулу, то можно ошибиться и получить неверные результаты. А это может отрицательно сказаться на работе электрического прибора.

О параллельном соединении резисторов: расчет подключения, определение мощности

Достаточно большое количество радиолюбителей занимается сборкой, модернизацией и ремонтом разнообразных схем; для кого-то это работа, а для кого-то просто увлечение или хобби. В любом случае необходимо иметь представление о процессах, происходящих в схеме, физических свойствах самих элементов цепи и особенностях взаимодействия элементов между собой.

Диагностика электронных схем

Компоненты электронных схем

Все множество компонентов и элементов делится на две основные группы:

  1. Активные элементы, особенностью которых является возможность усиливать проходящий по ним сигнал. К такой группе, в первую очередь, относятся транзисторы и построенные на их основе схемы;
  2. Пассивные элементы, которые не предназначены для усиления сигнала. Элементами, которые относятся к этой группе, являются резисторы, конденсаторы, катушки индуктивности, диоды и остальные аналоги данных компонентов.

Наиболее простым элементом по своим характеристикам и свойствам является резистор. Основное назначение резисторов заключается в ограничении величины тока, проходящего по нему. Все существующие резисторы подразделяются на два вида:

  • Постоянные – шунты, имеющие постоянное значение электрического сопротивления;
  • Переменные – шунты, сопротивление между контактами которого меняется механическим путем;

Центральными характеристиками резисторов являются:

  • Сила рассеивания, которая представляет собой максимальную мощность тока, выдерживаемую шунтом долгое время и рассеиваемую в виде тепла при постоянных характеристиках самого шунта;
  • Параметр точности представляет собой максимальное отступление от величины реального сопротивления в течение эксплуатации шунта;
  • Умение компонента противодействовать прохождению электротока в электрической цепи называется сопротивлением. Соответственно, с повышением сопротивления нарастает противостояние прохождению электротока.

Соединение резисторов

В радиотехнике встречается ряд конфигураций по состыковке компонентов в целом и шунтов в частности. Совмещение шунтов подразделяется на такие виды:

  • Набор параллельных резисторов;
  • Последовательное подключение шунтов;
  • Смешанное соединение резисторов.

Сопряжение резисторов

Последовательное соединение резисторов

Последовательным сопряжением компонентов называют такое сплочение нескольких шунтов, при котором каждый одиночный шунт подключается к следующему единичному шунту только в одной точке. Поскольку при последовательном включении по шунтам протекает одинаковый электрический ток, сталкивающийся постоянно с новой преградой в виде последующего сопротивления, то суммарное противодействие возрастает и равно сложению сопротивлений. Согласно картинке выше, при последовательном соединении суммарное последовательное сопротивление равно:

Rобщ=R1+R2, где:

  • Rобщ – суммарное противостояние цепи;
  • R1 – противодействие первого шунта;
  • R2 – противодействие второго шунта

Последовательное сопряжение шунтов

Напряжение при последовательном соединении компонентов понижается на любом отдельном элементе, опираясь на закон Ома, суммарное напряжение такого участка необходимо складывать. Соответственно, результирующий показатель напряжения возможно найти по выражению:

Uобщ=UR1+UR2, где:

  • Uобщ – суммарное напряжение участка;
  • UR1 – разность потенциалов на первом шунте;
  • UR2 – разность потенциалов на втором шунте.

Поскольку проходящий по компонентам электроток неизменный, то справедливо равенство:

Iобщ=IR1=IR2, где:

  • Iобщ – суммарная сила тока;
  • IR1 – электроток первого шунта;
  • IR2 – электроток второго шунта.

Дополнительная информация. При последовательной состыковке компонентов варьирование сопротивления любого элемента из этого участка влечет за собой как изменение сопротивления всего участка, так и изменение силы электротока этого участка.

Параллельное сопряжение резисторов

Параллельное соединение резисторов представляет собой такое соединение элементов, при котором резисторы подключаются друг к другу обоими выводами.

Дополнительная информация. В радиоэлектронике параллельные резисторы могут использоваться с целью снижения суммарного противодействия электротоку. Также мощность при параллельном стыковании компонентов возрастает сравнительно с каждым отдельным элементом.

При параллельном соединении шунтов через каждый единичный элемент потечет свой ток, и значение силы электротока будет обратно пропорционально сопротивлению компонента. Поскольку суммарная проводимость параллельного соединения возрастает, а общая сопротивляемость электротоку убывает, то, согласно закону Ома, общее сопротивление при параллельном соединении равняется:

  • Gобщ =1/Rобщ =1/R1+1/R2+1/R3;
  • Rобщ =1/Gобщ =R1R2R3/R1R2+R2R3+R1R3, где Gобщ – общая проводимость цепи.

Напряжение при параллельном соединении компонентов равняется разности потенциалов на каждом из компонентов:

Uобщ=UR1=UR2=UR3.

Направление токов в параллельной цепи

Калькулятор поможет определить суммарную силу тока цепи при параллельном совмещении, соответствующую сумме токов через каждый шунт:

Iобщ=IR1+IR2+IR3.

Смешанное соединение резисторов

Смешанное соединение резисторов представляет собой последовательное и параллельное соединение резисторов одновременно. Для определения суммарного противодействия цепи с разнотипной состыковкой шунтов требуется придерживаться последовательного алгоритма:

  1. Схематически разделить цепь на отдельные участки, включающие в себя последовательное и параллельное соединение сопротивлений;
  2. Рассчитать суммарный импеданс всех разделенных областей;
  3. Представить первоначальную схему в виде сопротивлений, имеющих некоторую величину эквивалентного сопротивления;
  4. Находите суммарное противодействие упрощенной схемы.

Понимание, что такое последовательное и параллельное соединение проводников, и поведения электрических характеристик при таком соединении позволит без особого труда проводить расчет, конструировать устройства различных конфигураций с требуемыми значениями параметров. Появится возможность упрощать и модернизировать схемы, вносить дополнительно какие-либо новшества в цепь.

Видео

Оцените статью:

Последовательное и параллельное соединение резисторов

Последовательное и параллельное соединение резисторов в схемах являются самыми распространенными, также – это база для расчета более сложных схем.

Последовательное подключение

Начнем с последовательного соединения. По этой схеме каждый резистор подключается с другим только в одной точке, их может быть в цепи 2, 3 и больше.

Рис. Последовательное подключение.
Обозначение:

Обозначим сопротивления: R1, R2, R3 и напряжение источника в цепи Uц. При подключении источника питания в ней начнет протекать ток Iц. В цепи с последовательным соединением ток протекает по всем резисторам один за другим. Поскольку ток течет через все резисторы их сопротивления и ток суммируется, Iц = I1+I2+I3, Rц = R1 +R2 + R3, чем больше отдельно взятое сопротивление, тем тяжелее электронам преодолевать участок цепи. Мощность резисторов при последовательном и параллельном соединении рассчитывается по разным формулам. В последовательных цепях – складываем, в параллельных – это обратно пропорциональная величина.

Параллельное соединение

Рис. Параллельное подключение.

Данный вид подключения характерен тем, что все элементы цепи соединяется выводами в одной точке друг другу, т.е. точка входа и выхода всех нагрузок сходятся в одну точку (или еще одно обозначение на схемах – //). Электроток, двигаясь по проводнику, дойдя до общего соединения делится на количество имеющихся веток. Если представить движение воды в трубе, то можно сказать, что вода двигающиеся по одной трубе, равномерно перетекает в несколько отводов, подсоединенных к ней. В нашем случае заряженные электроны, двигающиеся по проводнику, также растекаются на количества предложенных веток в узле.
Более наглядно это можно представить в виде формул:
1. Каждый вид соединения находится под одинаковым напряжением:
U = U1 = U2;
2. Суммарная сила тока равняется суммарному значению тока каждого участка
I = I1 + I2;
3. Сопротивление цепи равно сумме величина обратных сопротивлению участка:
1/R = 1/R1 + 17R2 + . . . + 1/Rn;
4. Сила тока пропорциональна сопротивлению каждого участка
I1/I2=R2/R1.
Далее рассмотрим схему как работает не только последовательное параллельное, но и смешанное соединение резисторов.

Смешанное подключение

Рис. Смешанное подключение резисторов

В электрических схемах используются не только типовые схемы, но и смешанное, созданное из критерий определенных требований. Чаще всего в схемах встречается третий вариант, представляющий набор из элементарных типов схем. В смешанных участках учитываются не только элементы, но и направления движения тока.
При вычислении мощности резисторов смешанного подключения используются формулы для параллельного и последовательного соединения резисторов, формула также является составной.

Основные законы электротехники, наиболее часто используемые для расчетов

Рассмотрим основные законы электротехники и свойства последовательного и параллельного соединения резисторов для участка цепи

Закон Ома

Напряжение находится по закону Ома по формуле I=U/R – чем больше сопротивление, тем меньше ток. Напряжение можно найти из этой же формулы. U=R*I, ток умножается на сопротивление. Запишем эту формулу для каждого участка U1=R1· I1, Un=Rn · In.

Законы Кирхгофа

Первый закон

Ещё один очень важный закон — это закон Кирхгофа. Для участка цепи постоянного тока их два.

Рис. иллюстрация к пояснению действия первого закона Кирхгофа.

Первый закон имеет формулировку: Сумма всех токов, входящих в узел и выходящих из него равна нулю.
Если посмотреть на схему, I1 – это ток, который заходит в узел, I2 и I3 – это электроны, которые вытекают из него.
Применяя формулировку первого закона можно записать формулу по-другому:
I1-I2+I3=0. В этой формуле знаки плюс имеют значения, которые прибывают в узел, минус, который отходит от него.

Второй закон Кирхгофа.
Рис. иллюстрация к пояснению действия второго закона Кирхгофа.

Если к цепи с включенными сопротивлениями подключен один источник ЭДС (батарея питания) тогда всё понятно, можно обойтись законом Ома. А, если, источников несколько и схема с различным схемным расположением элементов, тогда вступает в силу второй закон, который гласит: сумма токов всех источников питания для замкнутого контура, равна сумме падений напряжения на всех сопротивлениях участка в этом контуре.
E1- Е2 = – UR1 – UR2 или E1 = Е2 – UR1 – UR2.

Параллельное и последовательное соединение резисторов,  решение задач

Алгоритм расчёта смешанных подключений находится в тех же правилах, что и в элементарных схемах расчета последовательного и параллельного соединения резисторов. Ничего нового нет: нужно правильно разбить предложенную схему на пригодные для расчета участки. Участки, с элементами, подключены поочередно либо параллельно.

Рис. Порядок замещения при расчете сложных позиций более простыми.

Для решения задачи на последовательное и параллельное соединение резисторов необходимо правильно оценить цепи элементов. Рассмотрим схему №1 на рис.
На схеме присутствует параллельная и последовательная часть соединения элементов. Для расчета очень важно аккуратно, шаг за шагом упрощать цепи и не брать сразу всю схему (рис.1). Как же правильно определить параллельное и последовательное соединение резисторов?

Для примера расчета возьмем резисторы R3, R4, которые подключены параллельно. Эквивалентный резистор этих элементов, будет равенRэ. = 1/R34 =1/R3 + 1/R4, после преобразования формулы и приведения к одному знаменателю получим R34 = R3 · R4 / (R3 + R4). Э. = 1/3+1/4 /(3+4) =1,7 Ом.

Далее видно, что приведённая эквивалентное R эк и R6 соединены последовательно, чтобы узнать сопротивление их необходимо сложить, тогда общее сопротивление будет равно R346 = R34 + R6, тогда Rэк346 = 1,7 + 6 = 7, 7 Ом.
Заменяем на схеме одним общим элементом, теперь, позиция упрощается еще больше (рис 3).

Теперь образовалась ситуация – включение трех элементов в //. Как вычисляется такое соединение нам уже известно, 1/ R23465 = 1/ R2 +1/R346 + 1/R5 после вычисления правой части получаем 0,82 Ом. После окончательного вычисления получаем R23465 = 2,1 Ом. Здесь следует обратить внимание, что общее сопротивление получилось меньше самого меньшего из трех.

Заменяем эти сопротивление одним эквивалентным R23465. В конечном итоге все выглядит уже намного проще. Rц = Rэк + R1+ R2. R об. = R ц = 1,21 +7+1 =9,21 Ом.
Из приведенного алгоритма расчёта видно, как из сложной схемы путем простого математического вычисления и применения правил сокращения резисторов участок становится простой и понятной.

Схема с подключением сопротивлений «треугольником»
Рис. Расчетная схема соединения резисторов в треугольник.

Иногда некоторые затруднения возникают при разборе схемы соединения в треугольник.

Рассмотрим на примере рисунка расчет резисторов по этому подключению.
Из схемы видно, что R1 и R2 соединены последовательно Rэ12 будет соединяться R3 последовательно.

Затем Rэ123 соединяется с сопротивлением R4, R5 в последовательную цепь. Затем все это объединяется с Rэ в //.

Проведем несложные вычисления учитывая, что
R1, R2, R4, R5 равняется 1 Ом. R3, R7 – 2 Ом.
RЭ1,2 = R1+R2 = 1+1=2 Ом.

Вычисляем параллельное подключение: Rэ 12 с R3. Rэ1,3 = (Rэ12*R3) /(Rэ12+R3) = (2*2) /(2+2) = 1Ом.

Далее мы видим последовательное: RЭ123 + R4 + R5 = 1+1+1 = 3 Ом.
И последнее – Rэ123 4 5 с R6 – параллельное.

Общее сопротивление цепи Rц = Rоб = (RЭ1,2,3,4,5 *R6) /(RЭ1,2,3,4,5+R6) = (3 * 2) / (3+2) = 1,2 Ом. Как видно, что расчет подобного варианта также не сложный.

Расчет последовательного и параллельного подключения резисторов онлайн

Подсчитать значение мощность и сопротивлений подставляя их в формулы можно только в учебных целях, или, когда объемы не очень большие. Наиболее практичный вариант расчета является онлайн калькуляторы, которые расположены на многочисленных интернет ресурсах. Для расчёта любой сложности нужно правильно определить тип соединения резисторов последовательное или параллельное и внести данные для расчета в поля калькулятора.

Также такая форма расчета подойдет и для проверки результатов решения учебных задач.

Последовательное и параллельное соединение резисторов и конденсаторов

Электрические цепи состоят не только из резисторов, в них применяется большое количество различных деталей, например, конденсатор, которые подключаются в последовательное, // и смешанное соединение.

Рис. Замещения последовательно включенных элементов.

Определение этому элементу можно дать следующее: Конденсатор – это совокупность проводящих тел служащий для накопления электрического заряда.
Элементарный конденсатор имеет две пластины, форма этих пластин может быть различной: сферической, круглой, цилиндрической, прямоугольной – по форме пластин разделяется и тип конденсатора.

Важное свойство. Одно из важных свойств конденсатора: если заряжается одна пластина конденсатора, то благодаря явлению электростатической индукции заряжается и вторая половина, но с противоположным знаком.

Устройство конденсатора

Плоский конденсатор состоит из двух плоских пластин отстоящих друг от друга на маленькое расстояние. У конденсатора к двум пластинам припаивается вывод всего их получается два.

Типовые схемы подключения конденсаторов

Рассмотрим различные виды подключения конденсатора.

Последовательное

Первый вид — это последовательное соединение.
Предположим, что емкость этих конденсаторов будут равны. Тогда заряды также будут равны: q1=q2=q3, как и в примере с резисторами,  сложный тип позиций с конденсатором можно упростить, заменив несколько элементов одним. У элементов соединенных друг за другом, общая емкость будет обратно пропорциональная всем имеющимся элементам. То есть: Rэк будет равняться 1/С1 + 1/С2 +…. 1/Сn/
Напряжение складывается,  U эк = U1 + U2+ … Un.

Параллельное

Второй тип подключения конденсаторов – это соединение в паралель

Рис. Схема замещения элементов, включенных в параллель.

Соответственно эти конденсаторов обозначены C1, C2, … Cn заряды: Q1, Q2, … Qn и напряжение: U1, U2, … Un.

У элементов в // емкость складывается Сэ = C1 + C2 + … C n.
Напряжение Un на каждом конденсаторе будет равно напряжению на эквивалентном
Uэ = U1 = U2 =… = Un – это особенность параллельного подсоединения всех элементов цепи.

Емкость будет складываться из суммы отдельных элементов Сэ =С1 + С2 + … Сп.

Рис. Расчетные позиции элементов при различном включении.

Простая позиция, которая не требует преобразования №1 – последовательное подключение. По известной формуле для этих поз. запишем 1/Сэ = 1/С1 +1/С2 +1/С3,  подставив формулу значения, которые даны в условии задачи, получим 1/Сэ = 1/С1 +1/С2 +1/С3 = 59 мФ.

Не требует преобразования и 2 схема: емкость общего конденсатора будет равняться сумме конденсаторов которые включены в параллельной цепи: Сэ =С1 +С2 +С3
Сэ = 100 + 200 + 500 = 800 мФ.

Рассмотрев рис. №3 видно, что пара конденсаторов включена параллельно и один последовательно. Алгоритм преобразования таких цепей мы уже рассматривали, поэтому: сразу же находим емкость конденсатора Сэ соединения: Сэ = С1+С2 = 200+500 = 700 мФ.

Теперь находим общие эквивалентную емкость элементов с последовательным подключением 1/Сэ = 1/С2,3 +1/ С1 = 89 мф.

Практическая задача решена.

 Внимание покупателей подшипников

Уважаемые покупатели, отправляйте ваши вопросы и заявки по приобретению  подшипников и комплектующих на почту или звоните сейчас:

     +7(499)403 39 91  

   

  Доставка подшипников  по РФ  и зарубежью.

  Каталог подшипников на сайте themechanic.ru

 

 

Внимание покупателей подшипников

Уважаемые покупатели, отправляйте ваши вопросы и заявки по приобретению подшипников и комплектующих на почту или звоните сейчас:
tel:+7 (495) 646 00 12
[email protected]
Доставка подшипников по РФ и зарубежью.
Каталог подшипников на сайте

Внимание покупателей подшипников

Уважаемые покупатели, отправляйте ваши вопросы и заявки по приобретению подшипников и комплектующих на почту или звоните сейчас:
tel:+7 (495) 646 00 12
[email protected]
Доставка подшипников по РФ и зарубежью.
Каталог подшипников на сайте

Какие есть способы соединения приемников электрической энергии

Какие есть способы соединения приемников электрической энергии

Приемник электрической энергии — устройство, в котором происходит преобразование электрической энергии в другой вид энергии для ее использования.

При одновременном включении нескольких приемников электроэнергии в одну и ту же сеть, эти приемники можно легко рассматривать просто как элементы единой цепи, каждый из которых обладает собственным сопротивлением.

В ряде случаев такой подход оказывается вполне приемлемым: лампы накаливания, электрические обогреватели и т. п. — можно воспринимать как резисторы. То есть приборы можно заменить на их сопротивления, и легко произвести расчет параметров цепи.

Способ соединения приемников электроэнергии может быть одним из следующих: последовательный, параллельный или смешанный тип соединения.

Последовательное соединение

Когда несколько приемников (резисторов) соединяются в последовательную цепь, то есть второй вывод первого присоединяется к первому выводу второго, второй вывод второго соединяется с первым выводом третьего, второй вывод третьего с первым выводом четвертого и т. д., то при подключении такой цепи к источнику питания, через все элементы цепи потечет ток I одной и той же величины. Данную мысль поясняет приведенный рисунок.

Заменив приборы на их сопротивления, рисунок преобразуем в схему, тогда сопротивления с R1 по R4, соединенные последовательно, примут каждый на себя определенные напряжения, которые в сумме дадут значение ЭДС на зажимах источника питания. Для простоты здесь и далее изобразим источник в виде гальванического элемента.

Выразив падения напряжений через ток и через сопротивления, получим выражение для эквивалентного сопротивления последовательной цепи приемников: общее сопротивление последовательного соединения резисторов всегда равно алгебраической сумме всех сопротивлений, составляющих эту цепь. А поскольку напряжения на каждом из участков цепи можно найти из закона Ома (U = I*R, U1 = I*R1, U2 = I*R2 и т. д.) и E = U, то для нашей схемы получаем:

Напряжение на клеммах источника питания равно сумме падений напряжений на каждом из соединенных последовательно приемников, составляющих цепь.

Так как ток через всю цепь течет одного и того же значения, то справедливым будет утверждение, что напряжения на последовательно соединенных приемниках (резисторах) соотносятся между собой пропорционально сопротивлениям. И чем выше будет сопротивление, тем выше окажется и напряжение, приложенное к приемнику.

Для последовательного соединения резисторов в количестве n штук, обладающих одинаковыми сопротивлениями Rk, эквивалентное общее сопротивление цепи целиком будет в n раз больше каждого из этих сопротивлений: R = n*Rk. Соответственно и напряжения, приложенные к каждому из резисторов цепи будут между собой равны, и окажутся в n раз меньше напряжения, приложенного ко всей цепи: Uk = U/n.

Для последовательного соединения приемников электроэнергии характерны следующие свойства: если изменить сопротивление одного из приемников цепи, то напряжения на остальных приемниках цепи при этом изменятся; при обрыве одного из приемников ток прекратится во всей цепи, во всех остальных приемниках.

В силу этих особенностей последовательное соединение встречается редко, и используют его лишь там, где напряжение сети выше номинального напряжения приемников, в отсутствие альтернатив.

К примеру напряжением 220 вольт можно запитать две последовательно соединенные лампы равной мощности, каждая из которых рассчитана на напряжение 110 вольт. Ежели данные лампы при одинаковом номинальном напряжении питания будут обладать различной номинальной мощностью, то одна из них будет перегружена и скорее всего мгновенно перегорит.

Параллельное соединение

Параллельное соединение приемников предполагает включение каждого из них между парой точек электрической цепи с тем, чтобы они образовывали параллельные ветви, каждая из которых питается напряжением источника. Для наглядности опять заменим приемники их электрическими сопротивлениями, чтобы получить схему, по которой удобно вести расчет параметров.

Как уже было сказано, в случае параллельного соединения каждый из резисторов испытывает действие одного и того же напряжения. И в соответствии с законом Ома имеем: I1=U/R1, I2=U/R2, I3=U/R3.

Здесь I – ток источника. Первый закон Кирхгофа для данной цепи позволяет записать выражение для тока в неразветвленной ее части: I = I1+I2+I3.

Отсюда общее сопротивление для параллельного соединения между собой элементов цепи можно найти из формулы:

Величина обратная сопротивлению называется проводимостью G, и формулу для проводимости цепи, состоящей из нескольких параллельно соединенных элементов, также можно записать: G = G1 + G2 + G3. Проводимость цепи в случае параллельного соединения образующих ее резисторов равна алгебраической сумме проводимостей этих резисторов. Следовательно, при добавлении в цепь параллельных приемников (резисторов) суммарное сопротивление цепи уменьшится, а суммарная проводимость соответственно возрастет.

Токи в цепи состоящей из параллельно соединенных приемников, распределяются между ними прямо пропорционально их проводимостям, то есть обратно пропорционально их сопротивлениям. Здесь можно привести аналогию из гидравлики, где поток воды распределяется по трубам в соответствии с их сечениями, тогда большее сечение аналогично меньшему сопротивлению, то есть большей проводимости.

Если цепь состоит из нескольких (n) одинаковых резисторов, соединенных параллельно, то общее сопротивление цепи будет ниже в n раз, чем сопротивление одного из резисторов, а ток через каждый из резисторов будет меньше в n раз, чем общий ток: R = R1/n; I1 = I/n.

Цепь, состоящая из параллельно соединенных приемников, подключенная к источнику питания, отличается тем, что каждый из приемников находится под напряжением источника питания.

Для идеального источника электроэнергии справедливо утверждение: при подключении или отключении параллельно источнику резисторов, токи в остальных подключенных резисторах не изменятся, то есть при выходе из строя одного или нескольких приемников параллельной цепи, остальные будут продолжать работать в прежнем режиме.

В силу данных особенностей параллельное соединение обладает значительным преимуществом перед последовательным, и по этой причине именно соединение параллельное наиболее распространено в электрических сетях. Например, все электроприборы в наших домах предназначены для параллельного подключения к бытовой сети, и если отключить один, то остальным это ничуть не навредит.

Сравнение последовательных и параллельных цепей

Смешанное соединение

Под смешанным соединением приемников понимают такое их соединение, когда часть или несколько из них соединены между собой последовательно, а другая часть или несколько — параллельно. При этом вся цепь может быть образована из разных соединений таких частей между собой. Для примера рассмотрим схему:

Три последовательно соединенных резистора подключены к источнику питания, параллельно одному из них подключены еще два, а третий — параллельно всей цепи. Для нахождения полного сопротивления цепи идут путем последовательных преобразований: сложную цепь последовательно приводят к простому виду, последовательно вычисляя сопротивление каждого звена, и так находят общее эквивалентное сопротивление.

Для нашего примера. Сначала находят общее сопротивление двух резисторов R4 и R5, соединенных последовательно, затем сопротивление параллельного соединения их с R2, потом прибавляют к полученному значению R1 и R3, и после — вычисляют значение сопротивления всей цепи, включая параллельную ветвь R6.

Различные способы соединения приемников электроэнергии применяют на практике для различных целей, чтобы решать конкретные поставленные задачи. Например, смешанное соединение можно встретить в схемах плавного заряда электролитических конденсаторов в мощных блоках питания, где нагрузка (конденсаторы после диодного моста) сначала получает питание последовательно через резистор, затем резистор шунтируется контактами реле, и нагрузка оказывается подключенной к диодному мосту параллельно.

Ранее ЭлектроВести писали, что львовянка Оксана Денис подключила свою квартиру к солнечным панелям и ветроэнергетическим установкам.

По материалам: electrik.info.

Как отличается параллельное и последовательное соединение резисторов?

Большое разнообразие схем основано на двух видах соединений – последовательное параллельное. Для каждого типа существуют свои собственные законы и принципы. Именно это и позволяет создавать устройства с самыми различными техническими параметрами, в том числе и резисторы. Что же такое резистор?

Резистор – радиодеталь, созданная для контроля напряжения и тока в цепи, увеличивая либо понижая его. Резисторы могут быть двух видов – постоянные и переменные. Так, например, светодиоды требуют для себя совсем небольшого тока. Для этого в электрическую цепочку перед светодиодом устанавливается резистор, который обеспечивает необходимое напряжение для работы последнего.

В статье подробны рассмотрены все аспекты последовательного и параллельного подключения резисторов. Бонусом к статье являются видеоролик и детальная информационная статья на рассматриваемую тему.

Последовательное подключение

Начнем с последовательного соединения. По этой схеме каждый резистор подключается с другим только в одной точке, их может быть в цепи 2, 3 и больше. Обозначим сопротивления: R1, R2, R3 и напряжение источника в цепи Uц. При подключении источника питания в ней начнет протекать ток Iц. В цепи с последовательным соединением ток протекает по всем резисторам один за другим.

Поскольку ток течет через все резисторы их сопротивления и ток суммируется, Iц = I1+I2+I3, Rц = R1 +R2 + R3, чем больше отдельно взятое сопротивление, тем тяжелее электронам преодолевать участок цепи. Мощность резисторов при последовательном и параллельном соединении рассчитывается по разным формулам. В последовательных цепях — складываем, в параллельных — это обратно пропорциональная величина.

Последовательное соединение характеризуется тем, что элементы идут друг за другом. Конец одного подключается к началу другого. При подключении полученной цепочки к источнику тока получается кольцо.

Теоретическая часть

Последовательное соединение характерно тем, что через все элементы протекает ток одинаковой силы. То есть, если цепочка состоит из двух резисторов R1 и R2 (как на рисунке ниже), то ток протекающий через каждое из них и любую другую часть цепи будет одинаковой (I = I1 = I2). Суммарное сопротивление всей цепи последовательно соединенных резисторов считается как сумма сопротивлений всех ее элементов. То есть, номиналы складывают. R = R1 + R2 — это и есть формула расчета сопротивления при последовательном соединении резисторов. Если элементов больше двух, будет просто больше слагаемых. Еще одно свойство последовательного соединения — на каждом элементе напряжение отличается. Ток в цепи одинаковый, а напряжение на резисторе зависит от его номинала.

Последовательное подключение.

Примеры расчета

Давайте рассмотрим пример. Цепь представлена на рисунке выше. Есть источник тока и два сопротивления. Пусть R1=1,2 кОм, R2= 800 Ом, а ток в цепи 2 А. По закону Ома U = I * R. Подставляем наши значения:

  • U1 = R1 * I = 1200 Ом * 2 А = 2400 В;
  • U2 = R2 * I = 800 Ом * 2А = 1600 В.

Общее напряжение цепи считается как сумма напряжений на резисторах: U = U1 + U2 = 2400 В + 1600 В = 4000 В. Полученную цифру можно проверить. Для этого найдем суммарное сопротивление цепи и умножим его на ток.   R = R1 + R2 = 1200 Ом + 800 Ом = 2000 Ом.

Если подставить в формулу напряжения при последовательном соединении сопротивлений, получаем: U = R * I = 2000 Ом * 2  А = 4000 В. Получаем, что общее напряжение данной цепи 4000 В.

А теперь посмотрите на схему. На первом вольтметре (возле резистора R1) показания будут 2400 В, на втором  — 1600 В.  При этом напряжение источника питания — 4000 В. Последовательное соединение – это соединение двух или более резисторов в форме цепи, в которой каждый отдельный резистор соединяется с другим отдельным резистором только в одной точке.

Материал по теме: Как проверить варистор мультиметром.

Общее сопротивление R

общ

При таком соединении, через все резисторы проходит один и тот же электрический ток. Чем больше элементов на данном участке электрической цепи, тем «труднее» току протекать через него. Следовательно, при последовательном соединении резисторов их общее сопротивление увеличивается, и оно равно сумме всех сопротивлений.

Параллельное соединение резисторов

Параллельное соединение – это соединение, при котором резисторы соединяются между собой обоими контактами. В результате к одной точке (электрическому узлу) может быть присоединено несколько резисторов.

Параллельное подключение резисторов.

Общее сопротивление R

общ

При таком соединении, через каждый резистор потечет отдельный ток. Сила данного тока будет обратно пропорциональна сопротивлению резистора. В результате общая проводимость такого участка электрической цепи увеличивается, а общее сопротивление в свою очередь уменьшается. Таким образом, при параллельном подсоединении резисторов с разным сопротивлением, общее сопротивление будет всегда меньше значения самого маленького отдельного резистора. Если посмотреть на изображение параллельного соединения, заметно, что ко всем элементам прилагается одинаковое напряжение.

То есть, при параллельном соединении резисторов, на каждом из них будет одинаковое напряжение U = U1 = U2 = U3. Получается, что ток разделяется на несколько «ручейков». То есть, при параллельном соединении резисторов сила тока, протекающего через каждый из элементов, отличается. I = I1+I2+I3. И зависит сила тока (согласно тому же закону Ома) от сопротивления каждого участка цепи. В случае с параллельным соединением резисторов — от их номинала.

Предлагаем также почитать интересный материал про малоизвестные факты о двигателях постоянного тока в другой нашей статье.

Схема параллельного соединения

Общее сопротивление участка цепи при таком соединении становится ниже. Его высчитывают по формуле: 1/R = 1/R1 + 1/R + 1/R3+. Такая форма хоть и понятна, но неудобна. Формула расчета сопротивления параллельно подключенных резисторов получается тем сложнее, чем больше элементов соединены параллельно. Но больше двух-трех редко кто объединяет, так что на практике достаточно знать только две формулы приведенные ниже.

Если подставить значения в эти формулы, то заметим, что результат будет меньше, чем сопротивление резистора с наименьшим номиналом. Это стоит запомнить: результирующее сопротивление включенных параллельно резисторов будет ниже самого маленького номинала. Давайте сначала рассчитаем параллельное соединение двух резисторов разного номинала и посмотрим что получится.

Соединили параллельно 150 Ом и 100 Ом. Считаем результирующее: 150*100 / (150+100) = 15000/250 = 60 Ом. Если соединить 150 Ом и 50 Ом, получим: 150*50 / (150+50) = 7500 / 200 = 37,5 Ом. Как видим, в обоих случаях результат оказывается меньше чем самый низкий номинал соединенных деталей. Этим и пользуются, если в наличии нет сопротивления небольшого номинала. Проблема только в том, что подбирать сложновато: надо каждый раз считать используя калькулятор.

Как высчитывать сопротивление составных резисторов

Возможно, вам будет проще, если знать, что соединив два одинаковых резистора параллельно, получим результат в два раза меньше. Например, соединив параллельно два резистора по 100 Ом получим составное сопротивление 50 Ом. Проверим? Считаем: 100*100 / (100+100) = 10000 / 200 = 50 Ом. При соединении параллельно трех резисторов, считать приходится больше, так как формула сложнее.

Если подключить параллельно 150 Ом, 100 Ом и 50 Ом, результирующее будет 27,3 Ом. Попробуем с более низкими номиналами. Если параллельно включены 20 Ом, 15 Ом и 10 Ом. Получим результирующее сопротивление 4,61 Ом. Вот вам подтверждение правила. Суммарное сопротивление параллельно соединенных резисторов меньше чем самый низкий номинал.

Параллельное соединение резисторов

Параллельное соединение резисторов это соединение, в котором начала всех резисторов соединены в одну общую точку (А), а концы в другую общую точку. При этом по каждому резистору течет свой ток. При параллельном соединении при протекании тока из точки А в точку Б, он имеет несколько путей. Таким образом, увеличение числа параллельно соединенных резисторов ведет к увеличению путей протекания тока, то есть к уменьшению противодействия протеканию тока. А это значит, чем большее количество резисторов соединить параллельно, тем меньше станет значение общего сопротивления такого участка цепи. Общее сопротивление параллельно соединенных резисторов определяется следующим отношением: 1/Rобщ= 1/R1+1/R2+1/R3+…+1/Rn.

Формулы расчета параллельного и последовательного подключения.

Следует отметить, что здесь действует правило «меньше – меньшего». Это означает, что общее сопротивление всегда будет меньше сопротивления любого параллельно включенного резистора. Общее сопротивление для двух параллельно соединенных резисторов рассчитывается по следующей формуле: Rобщ= R1*R2/R1+R2.

Если имеет место два параллельно соединенных резистора с одинаковыми сопротивлениями, то их общее сопротивление будет равно половине сопротивления одного из них. Параллельное соединение резисторов. При параллельном соединении резисторов нескольких приемников они включаются между двумя точками электрической цепи, образуя параллельные ветви.

Заменяя лампы резисторами с сопротивлениями R1, R2, R3, При параллельном соединении ко всем резисторам приложено одинаковое напряжение U. Поэтому согласно закону Ома: I1=U/R1; I2=U/R2; I3=U/R3.

Ток в неразветвленной части цепи согласно первому закону Кирхгофа I = I1+I2+I3, или I = U / R1 + U / R2 + U / R3 = U (1/R1 + 1/R2 + 1/R3) = U / Rэк . Следовательно, эквивалентное сопротивление рассматриваемой цепи при параллельном соединении трех резисторов определяется формулой 1/Rэк = 1/R1 + 1/R2 + 1/R3. Вводя в формулу (24) вместо значений 1/Rэк, 1/R1, 1/R2 и 1/R3 соответствующие проводимости Gэк, G1, G2 и G3, получим: эквивалентная проводимость параллельной цепи равна сумме проводимостей параллельно соединенных резисторов:

Gэк = G1+ G2 +G3 (25)

Таким образом, при увеличении числа параллельно включаемых резисторов результирующая проводимость электрической цепи увеличивается, а результирующее сопротивление уменьшается. Из приведенных формул следует, что токи распределяются между параллельными ветвями обратно пропорционально их электрическим сопротивлениям или прямо пропорционально их проводимостям. Например, при трех ветвях I1 : I2 : I3 = 1/R1 : 1/R2 : 1/R3 = G1 + G2 + G3.

В этом отношении имеет место полная аналогия между распределением токов по отдельным ветвям и распределением потоков воды по трубам. Приведенные формулы дают возможность определить эквивалентное сопротивление цепи для различных конкретных случаев. Например, при двух параллельно включенных резисторах результирующее сопротивление цепи – Rэк=R1R2/(R1+R2) при трех параллельно включенных резисторах Rэк=R1R2R3/(R1R2+R2R3+R1R3).

При параллельном соединении нескольких, например n, резисторов с одинаковым сопротивлением R1 результирующее сопротивление цепи Rэк будет в n раз меньше сопротивления R1, т.е.

Rэк = R1 / n.

Проходящий по каждой ветви ток I1, в этом случае будет в п раз меньше общего тока: I1 = I / n. При параллельном соединении приемников, все они находятся под одним и тем же напряжением, и режим работы каждого из них не зависит от остальных. Это означает, что ток, проходящий по какому-либо из приемников, не будет оказывать существенного влияния на другие приемники. При всяком выключении или выходе из строя любого приемника остальные приемники остаются включенными.

Поэтому параллельное соединение имеет существенные преимущества перед последовательным, вследствие чего оно получило наиболее широкое распространение. В частности, электрические лампы и двигатели, предназначенные для работы при определенном (номинальном) напряжении, всегда включают параллельно.

На электровозах постоянного тока и некоторых тепловозах тяговые двигатели в процессе регулирования скорости движения нужно включать под различные напряжения, поэтому они в процессе разгона переключаются с последовательного соединения на параллельное.

Более подробную информацию можно узнать, прочитав файл по данной теме “Расчет мощности резисторов”.  Всю новую информацию по этой и многим другим темам, вы сможете найти в группе. Подписывайтесь на нашу группу в социальной сети «Вконтакте».

Для этого вам необходимо будет перейти по следующей ссылке https://vk.com/electroinfonet. Также в группе можно задавать вопросы и получать на них подробные ответы от профи. В завершение объемной статьи хочу выразить благодарность источникам, откуда мы черпали информацию:

www.elektroznatok.ru

www.themechanic.ru

www.electrono.ru

www.hightolow.ru

www.sxemotehnika.ru

Предыдущая

РезисторыКак рассчитать резистор для светодиода?

Следующая

РезисторыЧто такое фоторезистор?

Последовательное и параллельное подключение резисторов | РОБОТИП

Не так давно в статье было описано, как «разгадать» цветовую маркировку на резисторе. А в этой статье, говориться о том, как подобрать (рассчитать) резистор для светодиода. Сегодня будем говорить с вами о последовательном и параллельном подключении резисторов.

С резисторами все достаточно просто.

Последовательное подключение резисторов.

При последовательном подключении, резисторы следуют друг за другом:

Рис.1 — Последовательное подключение резисторов

Рис.1 — Последовательное подключение резисторов

При последовательном подключении вы увеличиваете общее сопротивление, поскольку электрическому току, в таком случае приходится преодолевать увеличенное препятствие.

На рис.1 мы видим резистор с сопротивлением 100 Ом и следующий за ним резистор тоже с сопротивлением 100 Ом (если смотреть слева на право — это и есть последовательное включение). Подключая их последовательно мы увеличиваем сопротивление.

Чтобы убедиться в том, что сопротивление увеличивается, давайте подадим напряжение 5В на нашу цепочку из резисторов и посмотрим токи идущие в нашей получившейся цепи при подключении одного и двух сопротивлений в отдельности. При подключении одного резистора номиналом 100 Ом:

Рис.2 — Ток идущий через сопротивление 100 Ом при 5В.

Рис.2 — Ток идущий через сопротивление 100 Ом при 5В.

Мы видим, что ток в замкнутой цепи по закону Ома будет равен:

I = U / R = 5 / 100 = 50 мА;

Подключим в нашу цепь еще один резистор на 100 Ом последовательно с первым резистором:

Рис.3 — Последовательное подключение резисторов.

Рис.3 — Последовательное подключение резисторов.

Мы можем видеть на рис.3, что ток в цепи вдвое меньше и равняется уже:

I = U / R = 5 / (100 + 100) = 25 мА;

Ток получился равным 25 мА, при подключении последовательно к нашему резистору еще одного резистора на 100 Ом.

Таким образом, мы наглядно видим, что последовательное подключение увеличивает общее сопротивление цепи, а с увеличением сопротивления уменьшается ток. Для наглядности и простоты расчета, резисторы были подобраны одного номинала. Но это не значит, что так будет всегда и везде. Резисторы могут быть любых номиналов и исполнения, суть останется такой же.

Параллельное соединение резисторов.

Если приложить два резистора боком друг к другу и соединить их выводы, то мы получим параллельное соединение.

Рис.4 — параллельное соединение резисторов

Рис.4 — параллельное соединение резисторов

Прибегнем к нашему трюку с источником питания на 5В. Подключим его к параллельному соединению из двух одинаковых резисторов:

Рис.5 — Параллельное соединение резисторов с одинаковым сопротивлением.

Рис.5 — Параллельное соединение резисторов с одинаковым сопротивлением.

Когда два резистора одного номинала подключены параллельно, то к каждому из них приложено полное напряжение равное 5В, при этом электрический ток в таком случаем проходит через эти резисторы одновременно. Поэтому при параллельном соединении двух одинаковых сопротивлений общее сопротивление находится просто — делится на два номинал одного из резисторов, а так как они равны мы имеем:

R = 100 / 2 = 50 Ом;

Такой упрощенный расчет, вытекает из формулы:

Rобщ = R1 * R2 / R1 + R2;

Если есть желание, можно подставить значения наших резисторов и получатся все те же 50 Ом. Но легче же просто поделить на 2 😉

Данное правило работает с резисторами соединенными параллельно у которых номинал сопротивления одинаковый. А что если соединяемые параллельно резисторы имеют разный номинал? В таком случае мы будем искать общее сопротивление только по формуле:

Rобщ = R1 * R2 / R1 + R2;

Причем количество резисторов может быть сколь угодно большим. В формуле добавятся R3, R4, R5 и т.д.

Для наглядности, давайте подключим к параллельно соединенным резисторам с разным значением сопротивления наше питание 5В и посмотрим значения тока в цепи и ветвях:

Рис. 6 — Ток в цепи и ветвях при параллельном соединении резисторов с разным значением сопротивления.

Рис. 6 — Ток в цепи и ветвях при параллельном соединении резисторов с разным значением сопротивления.

На рис.6 видно, как распределяется ток в общей цепи и ветвях. Для проверки рассчитаем по формуле общее сопротивление цепи, получим следующее:

Rобщ = R1 * R2 / R1 + R2 = 200 * 100 / 200 +100 = 20000 /300 = 66,67 Ом.

Теперь по закону Ома рассчитаем ток в общей цепи, чтобы сверить его с общим током в цепи на рис.6:

I = U / R = 5 / 66,67 = 0,075 А, что равно 75 мА.

Мы получили тоже значение общего тока в цепи, что и на рис.6, а значит общее сопротивление цепи равно 66, 67 Ом.

Итог следующий:

  • При последовательном соединении резисторов, общее сопротивление увеличивается;
  • При параллельном соединении резисторов, общее сопротивление уменьшается;

Подписывайтесь на канал РОБОТИП впереди много инетересного!

Параллельное соединение резисторов определение — Морской флот

Как правильно соединять резисторы?

О том, как соединять конденсаторы и рассчитывать их общую ёмкость уже рассказывалось на страницах сайта. А как соединять резисторы и посчитать их общее сопротивление? Именно об этом и будет рассказано в этой статье.

Резисторы есть в любой электронной схеме, причём их номинальное сопротивление может отличаться не в 2 – 3 раза, а в десятки и сотни раз. Так в схеме можно найти резистор на 1 Ом, и тут же неподалёку на 1000 Ом (1 кОм)!

Поэтому при сборке схемы либо ремонте электронного прибора может потребоваться резистор с определённым номинальным сопротивлением, а под рукой такого нет. В результате быстро найти подходящий резистор с нужным номиналом не всегда удаётся. Это обстоятельство тормозит процесс сборки схемы или ремонта. Выходом из такой ситуации может быть применение составного резистора.

Для того чтобы собрать составной резистор нужно соединить несколько резисторов параллельно или последовательно и тем самым получить нужное нам номинальное сопротивление. На практике это пригождается постоянно. Знания о правильном соединении резисторов и расчёте их общего сопротивления выручают и ремонтников, восстанавливающих неисправную электронику, и радиолюбителей, занятых сборкой своего электронного устройства.

Последовательное соединение резисторов.

В жизни последовательное соединение резисторов имеет вид:


Последовательно соединённые резисторы серии МЛТ

Принципиальная схема последовательного соединения выглядит так:

На схеме видно, что мы заменяем один резистор на несколько, общее сопротивление которых равно тому, который нам необходим.

Подсчитать общее сопротивление при последовательном соединении очень просто. Нужно сложить все номинальные сопротивления резисторов входящих в эту цепь. Взгляните на формулу.

Общее номинальное сопротивление составного резистора обозначено как Rобщ.

Номинальные сопротивления резисторов включённых в цепь обозначаются как R1, R2, R3,…RN.

Применяя последовательное соединение, стоит помнить одно простое правило:

Из всех резисторов, соединённых последовательно главную роль играет тот, у которого самое большое сопротивление. Именно он в значительной степени влияет на общее сопротивление.

Так, например, если мы соединяем три резистора, номинал которых равен 1, 10 и 100 Ом, то в результате мы получим составной на 111 Ом. Если убрать резистор на 100 Ом, то общее сопротивление цепочки резко уменьшиться до 11 Ом! А если убрать, к примеру, резистор на 10 Ом, то сопротивление будет уже 101 Ом. Как видим, резисторы с малыми сопротивлениями в последовательной цепи практически не влияют на общее сопротивление.

Параллельное соединение резисторов.

Можно соединять резисторы и параллельно:


Два резистора МЛТ-2, соединённых параллельно

Принципиальная схема параллельного соединения выглядит следующим образом:

Для того чтобы подсчитать общее сопротивление нескольких параллельно соединённых резисторов понадобиться знание формулы. Выглядит она вот так:

Эту формулу можно существенно упростить, если применять только два резистора. В таком случае формула примет вид:

Есть несколько простых правил, позволяющих без предварительного расчёта узнать, каково должно быть сопротивление двух резисторов, чтобы при их параллельном соединении получить то, которое требуется.

Если параллельно соединены два резистора с одинаковым сопротивлением, то общее сопротивление этих резисторов будет ровно в два раза меньше, чем сопротивление каждого из резисторов, входящих в эту цепочку.

Это правило исходит из простой формулы для расчёта общего сопротивления параллельной цепи, состоящей из резисторов одного номинала. Она очень проста. Нужно разделить номинальное сопротивление одного из резисторов на общее их количество:

Здесь R1 – номинальное сопротивление резистора. N – количество резисторов с одинаковым номинальным сопротивлением.

Ознакомившись с приведёнными формулами, вы скажите, что все они справедливы для расчёта ёмкости параллельно и последовательно соединённых конденсаторов. Да, только в отношении конденсаторов всё действует с точностью до «наоборот”. Узнать подробнее о соединении конденсаторов можно здесь.

Проверим справедливость показанных здесь формул на простом эксперименте.

Возьмём два резистора МЛТ-2 на 3 и 47 Ом и соединим их последовательно. Затем измерим общее сопротивление получившейся цепи цифровым мультиметром. Как видим оно равно сумме сопротивлений резисторов, входящих в эту цепочку.


Замер общего сопротивления при последовательном соединении

Теперь соединим наши резисторы параллельно и замерим их общее сопротивление.


Измерение сопротивления при параллельном соединении

Как видим, результирующее сопротивление (2,9 Ом) меньше самого меньшего (3 Ом), входящего в цепочку. Отсюда вытекает ещё одно известное правило, которое можно применять на практике:

При параллельном соединении резисторов общее сопротивление цепи будет меньше наименьшего сопротивления, входящего в эту цепь.

Что ещё нужно учитывать при соединении резисторов?

Во-первых, обязательно учитывается их номинальная мощность. Например, нам нужно подобрать замену резистору на 100 Ом и мощностью 1 Вт. Возьмём два резистора по 50 Ом каждый и соединим их последовательно. На какую мощность рассеяния должны быть рассчитаны эти два резистора?

Поскольку через последовательно соединённые резисторы течёт один и тот же постоянный ток (допустим 0,1 А), а сопротивление каждого из них равно 50 Ом, тогда мощность рассеивания каждого из них должна быть не менее 0,5 Вт. В результате на каждом из них выделится по 0,5 Вт мощности. В сумме это и будет тот самый 1 Вт.

Данный пример достаточно грубоват. Поэтому, если есть сомнения, стоит брать резисторы с запасом по мощности.

Подробнее о мощности рассеивания резистора читайте тут.

Во-вторых, при соединении стоит использовать однотипные резисторы, например, серии МЛТ. Конечно, нет ничего плохого в том, чтобы брать разные. Это лишь рекомендация.

При проектировании электрических схем возникает необходимость использования последовательного и параллельного соединений резисторов. Соединения применяются также и при ремонтах электрооборудования, поскольку в некоторых ситуациях невозможно найти эквивалентный номинал резистора. Выполнить расчет просто, и справиться с этой операцией может каждый.

Типы проводников

Проводимость веществом электрического тока связана с наличием в нем свободных носителей заряда. Их количество определяется по электронной конфигурации. Для этого необходима химическая формула вещества, при помощи которой можно вычислить их общее число. Значение для каждого элемента берется из периодической системы Дмитрия Ивановича Менделеева.

Электрический ток — упорядоченное движение свободных носителей заряда, на которые воздействует электромагнитное поле. При протекании тока по веществу происходит взаимодействие потока заряженных частиц с узлами кристаллической решетки, при этом часть кинетической энергии частицы превращается в тепловую энергию. Иными словами, частица «ударяется» об атом, а затем снова продолжает движение, набирая скорость под действием электромагнитного поля.

Процесс взаимодействия частиц с узлами кристаллической решетки называется электрической проводимостью или сопротивлением материала. Единицей измерения является Ом, а определить его можно при помощи омметра или расчитать. Согласно свойству проводимости, вещества можно разделить на 3 группы:

  1. Проводники (все металлы, ионизированный газ и электролитические растворы).
  2. Полупроводники (Si, Ge, GaAs, InP и InSb).
  3. Непроводники (диэлектрики или изоляторы).

Проводники всегда проводят электрический ток, поскольку содержат в своем атомарном строении свободные электроны, анионы, катионы и ионы. Полупроводники проводят электричество только при определенных условиях, которые влияют на наличие или отсутствие свободных электронов и дырок. К факторам, влияющим на проводимость, относятся следующие: температура, освещенность и т. д. Диэлектрики вообще не проводят электричество, поскольку в их структуре вообще отсутствуют свободные носители заряда. При выполнении расчетов каждый радиолюбитель должен знать зависимость сопротивления от некоторых физических величин.

Зависимость сопротивления

Значение электропроводимости зависит от нескольких факторов, которые необходимо учитывать при расчетах, изготовлении элементов резистивной нагрузки (резисторов), ремонте и проектировании устройств. К этим факторам необходимо отнести следующие:

  1. Температура окружающей среды и материала.
  2. Электрические величины.
  3. Геометрические свойства вещества.
  4. Тип материала, из которого изготовлен проводник (полупроводник).

К электрическим величинам можно отнести разность потенциалов (напряжение), электродвижущую силу (ЭДС) и силу тока. Геометрией проводника является его длина и площадь поперечного сечения.

Электрические величины

Зависимость величины электропроводимости от параметров электричества определяется законом Ома. Существует две формулировки: одна — для участка, а другая — для полной цепи. В первом случае соотношение определяются, исходя из значений силы тока (I) и напряжения (U) простой формулой: I = U / R. Из соотношения видна прямо пропорциональная зависимость тока от величины напряжения, а также обратно пропорциональная от сопротивления. Можно выразить R: R = U / I.

Для расчета электропроводимости всего участка следует воспользоваться соотношением между ЭДС (e), силой тока (i), а также внутренним сопротивлением источника питания (Rвн): i = e / (R+Rвн). В этом случае величина R вычисляется по формуле: R = (e / i) — Rвн. Однако при выполнении расчетов необходимо учитывать также геометрические параметры и тип проводника, поскольку они могут существенно повлиять на вычисления.

Тип и геометрические параметры

Свойство вещества к проводимости электричества определяется структурой кристаллической решетки, а также количеством свободных носителей. Исходя из этого, тип вещества является ключевым фактором, который определяет величину электропроводимости. В науке коэффициент, определяющий тип вещества, обозначается литерой «р» и называется удельным сопротивлением. Его значение для различных материалов (при температуре +20 градусов по Цельсию) можно найти в специальных таблицах.

Иногда для удобства расчетов используется обратная величина, которая называется удельной проводимостью (σ). Она связана с удельным сопротивлением следующим соотношением: p = 1 / σ. Площадь поперечного сечения (S) влияет на электрическое сопротивление. С физической точки зрения, зависимость можно понять следующим образом: при малом сечении происходят более частые взаимодействия частиц электрического тока с узлами кристаллической решетки. Поперечное сечение можно вычислить по специальному алгоритму:

  1. Измерение геометрических параметров проводника (диаметр или длину сторон) при помощи штангенциркуля.
  2. Визуально определить форму материала.
  3. Вычислить площадь поперечного сечения по формуле, найденной в справочнике или интернете.

В случае когда проводник имеет сложную структуру, необходимо вычислить величину S одного элемента, а затем умножить результат на количество элементов, входящих в его состав. Например, если провод является многожильным, то следует вычислить S для одной жилы. После этого нужно умножить, полученную величину S, на количество жил. Зависимость R от вышеперечисленных величин можно записать в виде соотношения: R = p * L / S. Литера «L» является длиной проводника. Однако для получения точных расчетов необходимо учитывать температурные показатели внешней среды и проводника.

Температурные показатели

Существует доказательство зависимости удельного сопротивления материала от температуры, основанное на физическом эксперименте. Для проведения опыта нужно собрать электрическую цепь, состоящую из следующих элементов: источника питания, нихромовой спирали, соединительных проводов амперметра и вольтметра. Приборы нужны для измерения значений силы тока и напряжения соответственно. При протекании электричества происходит нагревание нихромовой пружины. По мере ее нагревания, показания амперметра уменьшаются. При этом происходит существенное падение напряжения на участке цепи, о котором свидетельствуют показания вольтметра.

В радиотехнике уменьшение величины напряжение называется просадкой или падением. Формула зависимости р от температуры имеет следующий вид: p = p0 * [1 + a * (t — 20)]. Значение p0 — удельное сопротивление материала, взятого из таблицы, а литера «t» — температура проводника.

Температурный коэффициент «а» принимает следующие значения: для металлов — a>0, а для электролитических растворов — a Объединение резистивных радиокомпонентов

Для получения необходимого номинала сопротивления применяются два типа соединения резисторов: параллельное и последовательное. Если их соединить параллельно, то нужно два вывода одного резистора подключить к двум выводам другого. Если соединение является последовательным, то один вывод резистора соединяется с одним выводом другого резистора. Соединения используются для получения необходимых номиналов сопротивлений, а также для увеличения рассеивания мощности тока, протекающего по цепи.

Каждое из соединений обладает определенными характеристиками. Кроме того, последовательно или параллельно могут объединяться несколько резисторов. Соединения также могут быть смешанными, т. е. применяться оба типа объединения радиокомпонентов.

Параллельное соединение

При параллельном подключении значение напряжения на всех резисторах одинаковое, а сила тока — обратно пропорциональна их общему сопротивлению. В интернете web-разработчики создали для расчета величины общего сопротивления параллельного соединения резисторов онлайн-калькулятор.

Рассчитывается общее сопротивление при параллельном соединении по формуле: 1 / Rобщ = (1 / R1) + (1 / R2) + …+ (1 / Rn). Если выполнить математические преобразования и привести к общему знаменателю, то получится удобная формула параллельного соединения для расчета Rобщ. Она имеет следующий вид: Rобщ = (R1 * R2 * … * Rn) / (R1 + R2 + … + Rn). Если необходимо рассчитать величину Rобщ только для двух радиокомпонентов, то формула параллельного сопротивления имеет следующий вид: Rобщ = (R1 * R2) / (R1 + R2).

При ремонте или проектировании схемы устройства возникает задача объединения нескольких резистивных элементов для получения конкретной величины сопротивления. Например, значение Rобщ для определенной цепочки элементов равно 8 Ом, которое получено при расчетах. Перед радиолюбителем стоит задача, какие нужно подобрать номиналы для получения нужного значения (в стандартном ряду резисторов отсутствует радиокомпонент с номиналом в 8 Ом, а только 7,5 и 8,2). В этом случае нужно найти сопротивление при параллельном соединении резистивных элементов. Посчитать значение Rобщ для двух элементов можно следующим образом:

  1. Номинал резистора в 16 Ом подойдет.
  2. Подставить в формулу: R = (16 * 16) / (16 + 16) = 256 / 32 = 8 (Ом).

В некоторых случаях следует потратить больше времени на подбор необходимых номиналов. Можно применять не только два, но и три элемента. Сила тока вычисляется с использованием первого закона Кирхгофа. Формулировка закона следующая: общее значение тока, входящего и протекающего по цепи, равен выходному его значению. Величина силы тока для цепи, состоящей из двух резисторов (параллельное соединение) рассчитывается по такому алгоритму:

  1. Ток, протекающий через R1 и R2: I1 = U / R1 и I2 = U / R2 соответственно.
  2. Общий ток — сложение токов на резисторах: Iобщ = I1 + I2.

Например, если цепь состоит из 2 резисторов, соединенных параллельно, с номиналами в 16 и 7,5 Ом. Они запитаны от источника питания напряжением в 12 В. Значение силы тока на первом резисторе вычисляется следующим способом: I1 = 12 / 16 = 0,75 (А). На втором резисторе ток будет равен: I2 = 12 / 7,5 = 1,6 (А). Общий ток определяется по закону Кирхгофа: I = I1 + I2 = 1,6 + 0,75 = 2,35 (А).

Последовательное подключение

Последовательное включение резисторов также применяется в радиотехнике. Методы нахождения общего сопротивления, напряжения и тока отличаются от параллельного подключения. Основные правила соединения следующие:

  1. Ток не изменяется на участке цепи.
  2. Общее напряжение равно сумме падений напряжений на каждом резисторе.
  3. Rобщ = R1 + R2 + … + Rn.

Пример задачи следующий: цепочка, состоящая из 2 резисторов (16 и 7,5 Ом), питается от источника напряжением 12 В и током в 0,5 А. Необходимо рассчитать электрические параметры для каждого элемента. Порядок расчета следующий:

  1. I = I1 = I2 = 0,5 (А).
  2. Rобщ = R1 + R2 = 16 + 7,5 = 23,5 (Ом).
  3. Падения напряжения: U1 = I * R1 = 0,5 * 16 = 8 (В) и U2 = I * R2 = 0,5 * 7,5 = 3,75 (В).

Не всегда выполняется равенство напряжений (12 В не равно 8 + 3,75 = 11,75 В), поскольку при этом расчете не учитывается сопротивление соединительных проводов. Если схема является сложной, и в ней встречается два типа соединений, то нужно выполнять расчеты по участкам. В первую очередь, рассчитать для параллельного соединения, а затем для последовательного.

Таким образом, параллельное и последовательное соединения резисторов применяются для получения более точных значений сопротивлений, а также при отсутствии необходимого номинала радиокомпонента при проектировании или ремонте устройств.

Электрическое сопротивление характеризует свойство проводника препятствовать прохождению через него электрического тока. У каждого материала есть свое удельное сопротивление. Это табличная величина, и условно она считается постоянной.

Условно, потому что во многом эта характеристика зависит от внешних условий, например температуры. Сопротивление же какого-либо конкретного элемента (мы будем говорить о резисторах) складывается из многих факторов, например, из геометрических параметров, а когда речь идет о цепи переменного тока, то в расчеты включают также индуктивное и емкостное сопротивление, но об этом мы расскажем позже. Пока же — немного теории.

Закон Ома

В 1826 году немецкий физик Георг Ом на основе своих опытов вывел закон, согласно которому сила тока на участке цепи прямо пропорциональна напряжению, которое к нему приложено, и обратно пропорциональна сопротивлению участка. Из школьного курса мы знаем этот закон:

I=U/R

Позже он был сформулирован и для полной цепи:

I=ε/(R+r)

Где ε — ЭДС источника, R — сопротивление цепи, а r — сопротивление источника.

Мощность прибора

Электрический заряд при своем перемещении совершает работу.2/R

Последовательное и параллельное соединение

В реальной жизни мы редко имеем дело с одним проводником и одним источником. Достаточно взглянуть в любую принципиальную электрическую схему, например, такую простенькую:

(это схема микроволновки «Электроника»)

можно увидеть, что элементы в схеме соединены по-разному, но мы покажем вам базовые закономерности, которые работают в цепях.

Правила Кирхгофа

Если взять замкнутую электрическую цепь, по которой течет заряд, то можно определенно сказать: он никуда не денется. Сумма всех зарядов, которые текут в одной цепи, всегда одинакова. Это называется законом сохранения заряда, частным случаем общего закона сохранения (как говорится, если в одном месте что-то убудет, в другом непременно прибудет).

Отсюда мы и выводим тот факт, что в каждом узле цепи сумма токов равна нулю. То есть, если ток «приходит» в точку по ветке и «уходит» по двум — значит, первый равен сумме второго и третьего.

На этой картинке мы видим, что I1+I4=I2+I3

Это называется первым правилом Кирхгофа.

Если наша цепь не будет содержать узлов, значит, ток в ней будет величиной постоянной, а элементы, один за другим поставленные в цепь, будут давать падение напряжения. При этом общее напряжение в цепи останется тем же. Отсюда вытекает второе правило Кирхгофа: сумма напряжений на участках цепи будет равна ЭДС источников тока, входящий в эту цепь. Если у нас источник один, то будет верно равенство:

ε=U1+U2+U3+…+Un

Сумма падений напряжения будет, таким образом, нулевой.

В ситуациях, когда мы имеем дело с переменным током, падение будет наблюдаться на участках с конденсаторами и катушками — в цепях переменного тока у них появляется сопротивление (об этом позже).

Теперь, когда мы познакомились с теоретической частью, можем перейти к более приближенному к суровой реальности вопросу, а именно — расчету последовательного и параллельного соединения резисторов.

Примеры расчетов

Рассчитаем параметры цепей с разным типом соединения.

Как мы видим из рисунка, резисторы соединены один за другим, последовательным способом.2/R

Исходя из вышеперечисленных закономерностей, вы сможете рассчитывать самые причудливые соединения резисторов, можете попрактиковаться, взяв в библиотеке задачник.

Типы резисторов

Как уже было сказано ранее, элемент, который ставится в цепь для нагрузки, называется резистором. Ставят его для разных целей, главным образом для того, чтобы изменить тот или иной параметр на участке цепи. Например, понизить напряжение или силу тока, чтобы деталь, стоящая за резистором, не сгорела.

Предприятиями выпускается большой ассортимент таких изделий, и их можно по-разному классифицировать. Номинально резистор имеет то сопротивление, которое указано на нем, а по факту оно может зависеть от напряжения в сети (нелинейность), иметь разброс параметра (иногда до 20% доходит). По применяемой технологии резисторы можно разделить на:

  1. проволочные;
  2. композитные;
  3. металлофольговые;
  4. угольные;
  5. интегральные.

Фактическое сопротивление такого элемента может зависеть от температуры окружающей среды и даже от частоты, если мы имеем дело с переменным током. Дело в том, что часть ассортимента резисторов выполнены по проволочной технологии, то есть фактически они представляют собой мини-катушку. При малых частотах (50 Гц) это в расчет не берется, а вот на высоких (мегагерцы) паразитная индуктивность и индуктивное сопротивление может сказаться на работе схемы. Поэтому при выборе резистора для работы с высокочастотными схемами внимательно смотрите. по какой технологии он сделан. Отдайте предпочтение тонкослойным и композиционным изделиям.

Помимо этого, большое распространение получили переменные резисторы, значение сопротивления которых можно регулировать. Делается это чаще всего отверткой. Необходимость в таких изделиях продиктована разбросом параметров у обычных резисторов, а подстроечный вариант позволяет регулировать сопротивление.

Все вышесказанное актуально для цепей постоянного тока и переменного при невысоких частотах, и все это — при нормальных условиях внешней среды. Расчеты цепей при нарушении этих условий нуждаются в дополнительной корректировке: это связано с ограниченностью действия закона Ома. С чем связаны ограничения? Вот несколько примеров:

  1. при сверхнизких температурах многие проводники проявляют такое интересное явление, как сверхпроводимость;
  2. также сопротивление может разниться при нагревании;
  3. неприменим закон Ома для описания электрического тока в газах;
  4. наконец, обычный резистор можно просто пробить высоким напряжением.

Все это прекрасно работает. Не верите — можете поэкспериментировать у себя дома или провести замеры тестером. Например, изучить елочную гирлянду или показания счетчиков при включенных электроприборах (напомню, что в гирлянде лампочки соединены последовательно, а розетки в доме — параллельно). Удачи!

резисторов последовательно и параллельно

Цели обучения

К концу этого раздела вы сможете:

  • Нарисуйте цепь с резисторами, включенными параллельно и последовательно.
  • Рассчитайте падение напряжения тока на резисторе, используя закон Ома.
  • Contrast (Контраст), как рассчитывается общее сопротивление для резисторов, включенных последовательно и параллельно.
  • Объясните, почему полное сопротивление параллельной цепи меньше наименьшего сопротивления любого из резисторов в этой цепи.
  • Вычислить общее сопротивление цепи, которая содержит смесь резисторов, соединенных последовательно и параллельно.

Большинство схем имеет более одного компонента, называемого резистором , который ограничивает поток заряда в цепи. Мера этого предела расхода заряда называется сопротивлением . Простейшие комбинации резисторов — это последовательное и параллельное соединение, показанное на рисунке 1. Общее сопротивление комбинации резисторов зависит как от их индивидуальных значений, так и от способа их подключения.

Рис. 1. (a) Последовательное соединение резисторов. (б) Параллельное соединение резисторов.

Когда резисторы в серии ? Резисторы включены последовательно всякий раз, когда поток заряда, называемый током , должен проходить через устройства последовательно. Например, если ток течет через человека, держащего отвертку, в землю, тогда R 1 на Рисунке 1 (а) может быть сопротивлением вала отвертки, R 2 сопротивлением ее ручки , R 3 сопротивление тела человека и R 4 сопротивление его обуви.На рисунке 2 показаны резисторы, последовательно подключенные к источнику напряжения . Кажется разумным, что полное сопротивление является суммой отдельных сопротивлений, учитывая, что ток должен проходить через каждый резистор последовательно. (Этот факт был бы преимуществом для человека, желающего избежать поражения электрическим током, который мог бы уменьшить ток, надев обувь с резиновыми подошвами с высоким сопротивлением. прибор, уменьшающий рабочий ток.)

Рис. 2. Три резистора, подключенных последовательно к батарее (слева), и эквивалентное одиночное или последовательное сопротивление (справа).

Чтобы убедиться, что последовательно включенные сопротивления действительно складываются, давайте рассмотрим потерю электроэнергии, называемую падением напряжения , в каждом резисторе на рисунке 2. Согласно закону Ома , падение напряжения В на резистор, когда через него протекает ток, рассчитывается по формуле V = IR , где I равно току в амперах (A), а R — сопротивление в омах (Ω).Другой способ представить это: В, — это напряжение, необходимое для протекания тока I через сопротивление R . Таким образом, падение напряжения на R 1 составляет В 1 = IR 1 , что на R 2 составляет В 2 = IR 2 и что для R 3 это V 3 = IR 3 .Сумма этих напряжений равна выходному напряжению источника; то есть

В = В 1 + В 2 + В 3 .

Это уравнение основано на сохранении энергии и сохранении заряда. Электрическая потенциальная энергия может быть описана уравнением PE = qV , где q — электрический заряд, а V — напряжение. Таким образом, энергия, подаваемая источником, составляет кв.кв. , а энергия, рассеиваемая резисторами, равна

.

qV 1 + qV 2 + qV 3 .

Установление связей: законы сохранения

Вывод выражений для последовательного и параллельного сопротивления основан на законах сохранения энергии и сохранения заряда, которые гласят, что общий заряд и полная энергия постоянны в любом процессе. Эти два закона непосредственно участвуют во всех электрических явлениях и будут многократно использоваться для объяснения как конкретных эффектов, так и общего поведения электричества.

Эти энергии должны быть равны, потому что в цепи нет другого источника и другого назначения для энергии.Таким образом, qV = qV 1 + qV 2 + qV 3 . Плата q аннулируется, давая V = V 1 + V 2 + V 3 , как указано. (Обратите внимание, что одинаковое количество заряда проходит через батарею и каждый резистор за заданный промежуток времени, поскольку нет емкости для хранения заряда, нет места для утечки заряда и заряд сохраняется.) Теперь подстановка значений для отдельных напряжений дает

В = IR 1 + IR 2 + IR 3 = I ( R 1 + R 2 + R 3 ).

Обратите внимание, что для эквивалентного сопротивления одной серии R с , мы имеем

В = ИК с .

Это означает, что полное или эквивалентное последовательное сопротивление R с трех резисторов составляет R с = R 1 + R 2 + R 3 .Эта логика действительна в общем для любого количества резисторов, включенных последовательно; таким образом, полное сопротивление R с последовательного соединения составляет

R с = R 1 + R 2 + R 3 +…,

, как предлагается. Поскольку весь ток должен проходить через каждый резистор, он испытывает сопротивление каждого, а последовательно соединенные сопротивления просто складываются.

Пример 1. Расчет сопротивления, тока, падения напряжения и рассеиваемой мощности: анализ последовательной цепи

Предположим, что выходное напряжение батареи на рисунке 2 равно 12.0 В, а сопротивления равны R 1 = 1,00 Ом, R 2 = 6,00 Ом и R 3 = 13,0 Ом. а) Каково полное сопротивление? (б) Найдите ток. (c) Рассчитайте падение напряжения на каждом резисторе и покажите, как они складываются, чтобы равняться выходному напряжению источника. (d) Рассчитайте мощность, рассеиваемую каждым резистором. (e) Найдите выходную мощность источника и покажите, что она равна общей мощности, рассеиваемой резисторами.

Стратегия и решение для (а)

Общее сопротивление — это просто сумма отдельных сопротивлений, определяемая следующим уравнением:

[латекс] \ begin {array} {lll} {R} _ {\ text {s}} & = & {R} _ {1} + {R} _ {2} + {R} _ {3} \ \ & = & 1.00 \ text {} \ Omega + 6.00 \ text {} \ Omega + 13.0 \ text {} \ Omega \\ & = & 20.0 \ text {} \ Omega \ end {array} \\ [/ latex].

Стратегия и решение для (b)

Ток определяется по закону Ома: В = IR . Ввод значения приложенного напряжения и общего сопротивления дает ток для цепи:

[латекс] I = \ frac {V} {{R} _ {\ text {s}}} = \ frac {12.0 \ text {V}} {20.0 \ text {} \ Omega} = 0.60 \ text {A }\\[/латекс].

Стратегия и решение для (c)

Напряжение — или падение IR — на резисторе определяется законом Ома.Ввод тока и значения первого сопротивления дает

.

В 1 = IR 1 = (0,600 A) (1,0 Ом) = 0,600 В.

Аналогично

В 2 = IR 2 = (0,600 A) (6,0 Ом) = 3,60 В

и

V3 = IR 3 = (0,600 A) (13,0 Ом) = 7,80 В.

Обсуждение для (c)

Три капли IR добавляют к 12.0 В, прогноз:

В 1 + В 2 + В 3 = (0,600 + 3,60 + 7,80) В = 12,0 В.

Стратегия и решение для (d)

Самый простой способ рассчитать мощность в ваттах (Вт), рассеиваемую резистором в цепи постоянного тока, — это использовать закон Джоуля , P = IV , где P — электрическая мощность. В этом случае через каждый резистор протекает одинаковый полный ток.Подставляя закон Ома V = IR в закон Джоуля, мы получаем мощность, рассеиваемую первым резистором, как

P 1 = I 2 R 1 = (0,600 A) 2 (1,00 Ом) = 0,360 Вт

Аналогично

P 2 = I 2 R 2 = (0,600 A) 2 (6,00 Ом) = 2,16 Вт

и

P 3 = I 2 R 3 = (0.{2}} {R} \\ [/ latex], где В, — это падение напряжения на резисторе (а не полное напряжение источника). Будут получены те же значения.

Стратегия и решение для (e)

Самый простой способ рассчитать выходную мощность источника — использовать P = IV , где В, — напряжение источника. Это дает

P = (0,600 A) (12,0 В) = 7,20 Вт.

Обсуждение для (e)

Обратите внимание, что по совпадению общая мощность, рассеиваемая резисторами, также равна 7.20 Вт, столько же, сколько мощность, выдаваемая источником. То есть

P 1 + P 2 + P 3 = (0,360 + 2,16 + 4,68) W = 7,20 Вт.

Мощность — это энергия в единицу времени (ватт), поэтому для сохранения энергии требуется, чтобы выходная мощность источника была равна общей мощности, рассеиваемой резисторами.

Основные характеристики резисторов серии

  1. Последовательные сопротивления добавить: R с = R 1 + R 2 + R 3 +….
  2. Одинаковый ток протекает последовательно через каждый резистор.
  3. Отдельные последовательно включенные резисторы не получают полное напряжение источника, а делят его.

На рисунке 3 показаны резисторы , включенные параллельно , подключенные к источнику напряжения. Резисторы включены параллельно, когда каждый резистор подключен непосредственно к источнику напряжения с помощью соединительных проводов с незначительным сопротивлением. Таким образом, к каждому резистору приложено полное напряжение источника. Каждый резистор потребляет такой же ток, как если бы он один был подключен к источнику напряжения (при условии, что источник напряжения не перегружен).Например, автомобильные фары, радио и т. Д. Подключены параллельно, так что они используют полное напряжение источника и могут работать полностью независимо. То же самое и в вашем доме, или в любом другом здании. (См. Рисунок 3 (b).)

Рис. 3. (a) Три резистора, подключенных параллельно батарее, и эквивалентное одиночное или параллельное сопротивление. (б) Электроснабжение в доме. (Источник: Dmitry G, Wikimedia Commons)

Чтобы найти выражение для эквивалентного параллельного сопротивления R p , давайте рассмотрим протекающие токи и их связь с сопротивлением.Поскольку каждый резистор в цепи имеет полное напряжение, токи, протекающие через отдельные резисторы, равны [латекс] {I} _ {1} = \ frac {V} {{R} _ {1}} \\ [/ latex] , [латекс] {I} _ {2} = \ frac {V} {{R} _ {2}} \\ [/ latex] и [латекс] {I} _ {3} = \ frac {V} {{R} _ {3}} \\ [/ латекс]. Сохранение заряда подразумевает, что полный ток I , производимый источником, является суммой этих токов:

I = I 1 + I 2 + I 3 .

Подстановка выражений для отдельных токов дает

[латекс] I = \ frac {V} {{R} _ {1}} + \ frac {V} {{R} _ {2}} + \ frac {V} {{R} _ {3}} = V \ left (\ frac {1} {{R} _ {1}} + \ frac {1} {{R} _ {2}} + \ frac {1} {{R} _ {3}} \ справа) \\ [/ латекс].

Обратите внимание, что закон Ома для эквивалентного одиночного сопротивления дает

[латекс] I = \ frac {V} {{R} _ {p}} = V \ left (\ frac {1} {{R} _ {p}} \ right) \\ [/ latex].

Члены в круглых скобках в последних двух уравнениях должны быть равны. Обобщая для любого количества резисторов, общее сопротивление R p параллельного соединения связано с отдельными сопротивлениями на

[латекс] \ frac {1} {{R} _ {p}} = \ frac {1} {{R} _ {1}} + \ frac {1} {{R} _ {2}} + \ гидроразрыв {1} {{R} _ {\ text {.} 3}} + \ text {.} \ Text {…} \\ [/ latex]

Это соотношение приводит к общему сопротивлению R p , которое меньше наименьшего из отдельных сопротивлений. (Это видно в следующем примере.) При параллельном подключении резисторов от источника течет больше тока, чем протекает по любому из них по отдельности, поэтому общее сопротивление ниже.

Пример 2. Расчет сопротивления, тока, рассеиваемой мощности и выходной мощности: анализ параллельной цепи

Пусть выходное напряжение батареи и сопротивления в параллельном соединении на Рисунке 3 будут такими же, как в ранее рассмотренном последовательном соединении: В = 12.0 В, R 1 = 1,00 Ом, R 2 = 6,00 Ом и R 3 = 13,0 Ом. а) Каково полное сопротивление? (б) Найдите полный ток. (c) Рассчитайте токи в каждом резисторе и покажите, как они складываются, чтобы равняться общему выходному току источника. (d) Рассчитайте мощность, рассеиваемую каждым резистором. (e) Найдите выходную мощность источника и покажите, что она равна общей мощности, рассеиваемой резисторами.

Стратегия и решение для (а)

Общее сопротивление для параллельной комбинации резисторов находится с помощью следующего уравнения.Ввод известных значений дает

[латекс] \ frac {1} {{R} _ {p}} = \ frac {1} {{R} _ {1}} + \ frac {1} {{R} _ {2}} + \ frac {1} {{R} _ {3}} = \ frac {1} {1 \ text {.} \ text {00} \ text {} \ Omega} + \ frac {1} {6 \ text {. } \ text {00} \ text {} \ Omega} + \ frac {1} {\ text {13} \ text {.} 0 \ text {} \ Omega} \\ [/ latex].

Таким образом,

[латекс] \ frac {1} {{R} _ {p}} = \ frac {1.00} {\ text {} \ Omega} + \ frac {0 \ text {.} \ Text {1667}} {\ текст {} \ Omega} + \ frac {0 \ text {.} \ text {07692}} {\ text {} \ Omega} = \ frac {1 \ text {.} \ text {2436}} {\ text { } \ Omega} \\ [/ латекс].

(Обратите внимание, что в этих вычислениях каждый промежуточный ответ отображается с дополнительной цифрой.) Мы должны перевернуть это, чтобы найти полное сопротивление R p . Это дает

[латекс] {R} _ {\ text {p}} = \ frac {1} {1 \ text {.} \ Text {2436}} \ text {} \ Omega = 0 \ text {.} \ Text { 8041} \ text {} \ Omega \\ [/ latex].

Общее сопротивление с правильным количеством значащих цифр составляет R p = 0,804 Ом

Обсуждение для (а)

R p , как и предполагалось, меньше наименьшего индивидуального сопротивления.

Стратегия и решение для (b)

Полный ток можно найти из закона Ома, заменив полное сопротивление R p . Это дает

[латекс] I = \ frac {V} {{R} _ {\ text {p}}} = \ frac {\ text {12.0 V}} {0.8041 \ text {} \ Omega} = \ text {14} \ text {.} \ text {92 A} \\ [/ latex].

Обсуждение для (б)

Ток I для каждого устройства намного больше, чем для тех же устройств, подключенных последовательно (см. Предыдущий пример).Схема с параллельным соединением имеет меньшее общее сопротивление, чем резисторы, включенные последовательно.

Стратегия и решение для (c)

Отдельные токи легко вычислить по закону Ома, поскольку каждый резистор получает полное напряжение. Таким образом,

[латекс] {I} _ {1} = \ frac {V} {{R} _ {1}} = \ frac {12.0 \ text {V}} {1.00 \ text {} \ Omega} = 12.0 \ text {A} \\ [/ латекс].

Аналогично

[латекс] {I} _ {2} = \ frac {V} {{R} _ {2}} = \ frac {12.0 \ text {V}} {6.00 \ text {} \ Omega} = 2 \ text {.} \ text {00} \ text {A} \\ [/ latex]

и

[латекс] {I} _ {3} = \ frac {V} {{R} _ {3}} = \ frac {\ text {12} \ text {.} 0 \ text {V}} {\ text {13} \ text {.} \ Text {0} \ text {} \ Omega} = 0 \ text {.} \ Text {92} \ text {A} \\ [/ latex].

Обсуждение для (c)

Общий ток складывается из отдельных токов:

I 1 + I 2 + I 3 = 14,92 A.

Это соответствует сохранению заряда.{2}} {13.0 \ text {} \ Omega} = 11.1 \ text {W} \\ [/ latex].

Обсуждение для (d)

Мощность, рассеиваемая каждым резистором при параллельном подключении, значительно выше, чем при последовательном подключении к тому же источнику напряжения.

Стратегия и решение для (e)

Общую мощность также можно рассчитать несколькими способами. Выбрав P = IV и введя полный ток, получим

P = IV = (14,92 A) (12,0 В) = 179 Вт.

Обсуждение для (e)

Общая мощность, рассеиваемая резисторами, также составляет 179 Вт:

P 1 + P 2 + P 3 = 144 Вт + 24,0 Вт + 11,1 Вт = 179 Вт

Это соответствует закону сохранения энергии.

Общее обсуждение

Обратите внимание, что как токи, так и мощность при параллельном подключении больше, чем для тех же устройств, подключенных последовательно.

Основные характеристики резисторов, подключенных параллельно
  1. Параллельное сопротивление определяется из [latex] \ frac {1} {{R} _ {\ text {p}}} = \ frac {1} {{R} _ {1}} + \ frac {1} { {R} _ {2}} + \ frac {1} {{R} _ {3}} + \ text {…} \\ [/ latex], и оно меньше любого отдельного сопротивления в комбинации.
  2. На каждый параллельно включенный резистор подается такое же полное напряжение источника. (В системах распределения электроэнергии чаще всего используются параллельные соединения для питания бесчисленных устройств, обслуживаемых одним и тем же напряжением, и для того, чтобы они могли работать независимо.)
  3. Параллельные резисторы не получают суммарный ток каждый; они делят это.

Сочетания последовательного и параллельного

Более сложные соединения резисторов иногда представляют собой просто комбинации последовательного и параллельного. Они часто встречаются, особенно если учесть сопротивление провода. В этом случае сопротивление провода включено последовательно с другими сопротивлениями, включенными параллельно. Комбинации последовательного и параллельного подключения можно свести к одному эквивалентному сопротивлению, используя технику, показанную на рисунке 4.Различные части идентифицируются как последовательные или параллельные, уменьшаются до их эквивалентов и далее уменьшаются до тех пор, пока не останется единственное сопротивление. Процесс более трудоемкий, чем трудный.

Рис. 4. Эта комбинация из семи резисторов имеет как последовательные, так и параллельные части. Каждое из них идентифицируется и приводится к эквивалентному сопротивлению, а затем уменьшается до тех пор, пока не будет достигнуто единичное эквивалентное сопротивление.

Самая простая комбинация последовательного и параллельного сопротивления, показанная на рисунке 4, также является наиболее поучительной, поскольку она используется во многих приложениях.Например, R 1 может быть сопротивлением проводов от автомобильного аккумулятора к его электрическим устройствам, которые подключены параллельно. R 2 и R 3 могли быть стартером и светом салона. Ранее мы предполагали, что сопротивление провода незначительно, но, когда это не так, оно имеет важные последствия, как показывает следующий пример.

Пример 3. Расчет сопротивления,

IR Падение, ток и рассеиваемая мощность: объединение последовательных и параллельных цепей

На рис. 5 показаны резисторы из двух предыдущих примеров, подключенные другим способом — комбинацией последовательного и параллельного подключения.Можно считать R 1 сопротивлением проводов, ведущих к R 2 и R 3 . (а) Найдите полное сопротивление. (b) Что такое падение IR в R 1 ? (c) Найдите текущие значения от I 2 до R 2 . (d) Какую мощность рассеивает R 2 ?

Рис. 5. Эти три резистора подключены к источнику напряжения, так что R 2 и R 3 параллельны друг другу, и эта комбинация включена последовательно с R 1 .

Стратегия и решение для (а)

Чтобы найти полное сопротивление, отметим, что R 2 и R 3 находятся параллельно, и их комбинация R p находится последовательно с R 1 . Таким образом, полное (эквивалентное) сопротивление этой комбинации составляет

.

R до = R 1 + R p .

Сначала находим R p , используя уравнение для параллельных резисторов и вводя известные значения:

[латекс] \ frac {1} {{R} _ {\ text {p}}} = \ frac {1} {{R} _ {2}} + \ frac {1} {{R} _ {3 }} = \ frac {1} {6 \ text {.} \ text {00} \ text {} \ Omega} + \ frac {1} {\ text {13} \ text {.} 0 \ text {} \ Omega} = \ frac {0.2436} {\ text {} \ Омега} \\ [/ латекс].

Инвертирование дает

[латекс] {R} _ {\ text {p}} = \ frac {1} {0,2436} \ text {} \ Omega = 4.11 \ text {} \ Omega \\ [/ latex].

Таким образом, общее сопротивление равно

.

R до = R 1 + R p = 1,00 Ом + 4,11 Ом = 5,11 Ом.

Обсуждение для (а)

Общее сопротивление этой комбинации является промежуточным между значениями чистой серии и чистой параллели (20.0 Ом и 0,804 Ом соответственно), найденные для тех же резисторов в двух предыдущих примерах.

Стратегия и решение для (b)

Чтобы найти падение IR в R 1 , отметим, что полный ток I протекает через R 1 . Таким образом, его падение IR составляет

В 1 = ИК 1

Мы должны найти I , прежде чем сможем вычислить V 1 .Полный ток I находится с помощью закона Ома для схемы. То есть

[латекс] I = \ frac {V} {{R} _ {\ text {tot}}} = \ frac {\ text {12.0} \ text {V}} {5.11 \ text {} \ Omega} = 2.35 \ text {A} \\ [/ latex].

Вводя это в выражение выше, мы получаем

В 1 = IR 1 = (2,35 A) (1,00 Ом) = 2,35 В.

Обсуждение для (б)

Напряжение, приложенное к R 2 и R 3 , меньше полного напряжения на величину В 1 .Когда сопротивление провода велико, это может существенно повлиять на работу устройств, представленных R 2 и R 3 .

Стратегия и решение для (c)

Чтобы найти ток через R 2 , мы должны сначала найти приложенное к нему напряжение. Мы называем это напряжение В p , потому что оно приложено к параллельной комбинации резисторов. Напряжение, приложенное как к R 2 , так и к R 3 , уменьшается на величину В 1 , и поэтому оно составляет

В p = V V 1 = 12.0 В — 2,35 В = 9,65 В.

Теперь ток I 2 через сопротивление R 2 находится по закону Ома:

[латекс] {I} _ {2} = \ frac {{V} _ {\ text {p}}} {{R} _ {2}} = \ frac {9.65 \ text {V}} {6.00 \ текст {} \ Omega} = 1,61 \ text {A} \\ [/ latex].

Обсуждение для (c)

Ток меньше 2,00 А, которые протекали через R 2 , когда он был подключен параллельно к батарее в предыдущем примере параллельной цепи.

Стратегия и решение для (d)

Мощность, рассеиваемая R 2 равна

P 2 = ( I 2 ) 2 R 2 = (1,61 A) 2 (6,00 Ом) = 15,5 Вт

Обсуждение для (d)

Мощность меньше 24,0 Вт, рассеиваемых этим резистором при параллельном подключении к источнику 12,0 В.

Одним из следствий этого последнего примера является то, что сопротивление в проводах снижает ток и мощность, подаваемую на резистор.Если сопротивление провода относительно велико, как в изношенном (или очень длинном) удлинителе, то эти потери могут быть значительными. Если потребляется большой ток, падение IR в проводах также может быть значительным.

Например, когда вы роетесь в холодильнике и включается мотор, свет холодильника на мгновение гаснет. Точно так же вы можете увидеть тусклый свет в салоне, когда вы запускаете двигатель вашего автомобиля (хотя это может быть связано с сопротивлением внутри самой батареи).

То, что происходит в этих сильноточных ситуациях, показано на рисунке 6. Устройство, обозначенное как R 3 , имеет очень низкое сопротивление, поэтому при его включении протекает большой ток. Этот увеличенный ток вызывает большее падение IR в проводах, представленных R 1 , уменьшая напряжение на лампе (которое составляет R 2 ), которое затем заметно гаснет.

Рис. 6. Почему гаснет свет, когда включен большой прибор? Ответ заключается в том, что большой ток, потребляемый двигателем прибора, вызывает значительное падение напряжения в проводах и снижает напряжение на свету.

Проверьте свое понимание

Можно ли любую произвольную комбинацию резисторов разбить на последовательную и параллельную? Посмотрите, сможете ли вы нарисовать принципиальную схему резисторов, которые нельзя разбить на комбинации последовательно и параллельно.

Решение Нет, есть много способов подключения резисторов, которые не являются комбинациями последовательного и параллельного, включая петли и переходы. В таких случаях правила Кирхгофа, которые будут включены в Правила Кирхгофа, позволят вам проанализировать схему.

Стратегии решения проблем для последовательных и параллельных резисторов
  1. Нарисуйте четкую принципиальную схему, обозначив все резисторы и источники напряжения. Этот шаг включает список известных проблем, поскольку они отмечены на вашей принципиальной схеме.
  2. Определите, что именно необходимо определить в проблеме (определите неизвестные). Письменный список полезен.
  3. Определите, включены ли резисторы последовательно, параллельно или в комбинации последовательно и параллельно.Изучите принципиальную схему, чтобы сделать эту оценку. Резисторы включены последовательно, если через них должен последовательно проходить один и тот же ток.
  4. Используйте соответствующий список основных функций для последовательных или параллельных подключений, чтобы найти неизвестные. Есть один список для серий, а другой — для параллелей. Если ваша проблема представляет собой комбинацию последовательного и параллельного соединения, уменьшайте ее поэтапно, рассматривая отдельные группы последовательных или параллельных соединений, как это сделано в этом модуле и примерах. Особое примечание: при обнаружении R необходимо проявлять осторожность.
  5. Проверьте, являются ли ответы разумными и последовательными. Единицы и числовые результаты должны быть разумными. Общее последовательное сопротивление должно быть больше, а общее параллельное сопротивление, например, должно быть меньше. Мощность должна быть больше для одних и тех же устройств, подключенных параллельно, по сравнению с последовательными и т. Д.

Сводка раздела

Концептуальные вопросы

1. Переключатель имеет переменное сопротивление, близкое к нулю в замкнутом состоянии и очень большое в разомкнутом, и он включен последовательно с устройством, которым он управляет.Объясните влияние переключателя на рис. 7 на ток в разомкнутом и замкнутом состоянии.

Рис. 7. Переключатель обычно включается последовательно с источником сопротивления и напряжения. В идеале переключатель имеет почти нулевое сопротивление в замкнутом состоянии, но имеет чрезвычайно большое сопротивление в разомкнутом состоянии. (Обратите внимание, что на этой диаграмме скрипт E представляет напряжение (или электродвижущую силу) батареи.)

2. Какое напряжение на разомкнутом переключателе на Рисунке 7?

3. На разомкнутом переключателе есть напряжение, как на Рисунке 7.Почему же тогда мощность, рассеиваемая разомкнутым переключателем, мала?

4. Почему мощность, рассеиваемая замкнутым переключателем, как на Рисунке 7, мала?

5. Студент в физической лаборатории по ошибке подключил электрическую лампочку, батарею и выключатель, как показано на рисунке 8. Объясните, почему лампочка горит, когда выключатель разомкнут, и гаснет, когда он замкнут. (Не пытайтесь — батарея сильно разряжается!)

Рис. 8. Ошибка подключения. Включите этот переключатель параллельно устройству, обозначенному [латекс] R [/ латекс].(Обратите внимание, что на этой диаграмме скрипт E представляет напряжение (или электродвижущую силу) батареи.)

6. Зная, что сила электрического шока зависит от величины тока, протекающего через ваше тело, вы бы предпочли, чтобы он был включен последовательно или параллельно с сопротивлением, таким как нагревательный элемент тостера, если он шокирован им? Объяснять.

7. Были бы ваши фары тусклыми при запуске двигателя автомобиля, если бы провода в вашем автомобиле были сверхпроводниками? (Не пренебрегайте внутренним сопротивлением батареи.) Объяснять.

8. Некоторые гирлянды праздничных огней соединены последовательно для экономии затрат на проводку. В старой версии использовались лампочки, которые при перегорании прерывали электрическое соединение, как открытый выключатель. Если одна такая лампочка перегорит, что случится с остальными? Если такая цепочка работает от 120 В и имеет 40 одинаковых лампочек, каково нормальное рабочее напряжение каждой? В более новых версиях используются лампы, которые при перегорании замыкаются накоротко, как замкнутый выключатель. Если одна такая лампочка перегорит, что случится с остальными? Если такая цепочка работает от 120 В и в ней осталось 39 идентичных лампочек, каково тогда рабочее напряжение каждой?

9.Если две бытовые лампочки мощностью 60 и 100 Вт подключить последовательно к бытовой электросети, какая из них будет ярче? Объяснять.

10. Предположим, вы проводите физическую лабораторию, в которой вас просят вставить резистор в цепь, но все прилагаемые резисторы имеют большее сопротивление, чем запрошенное значение. Как бы вы соединили доступные сопротивления, чтобы попытаться получить меньшее запрошенное значение?

11. Перед Второй мировой войной некоторые радиостанции получали питание через «шнур сопротивления», который имел значительное сопротивление.Такой резистивный шнур снижает напряжение до желаемого уровня для ламп радиоприемника и т.п., и это экономит расходы на трансформатор. Объясните, почему шнуры сопротивления нагреваются и тратят энергию при включенном радио.

12. У некоторых лампочек есть три уровня мощности (не включая ноль), получаемые от нескольких нитей накала, которые индивидуально переключаются и соединяются параллельно. Какое минимальное количество нитей нити необходимо для трех режимов мощности?

Задачи и упражнения

Примечание. Можно считать, что данные, взятые из цифр, имеют точность до трех значащих цифр.

1. (а) Каково сопротивление десяти последовательно соединенных резисторов сопротивлением 275 Ом? (б) Параллельно?

2. (a) Каково сопротивление последовательно соединенных резисторов 1,00 × 10 2 Ом, 2,50 кОм и 4,00 кОм? (б) Параллельно?

3. Какое наибольшее и наименьшее сопротивление можно получить, соединив резисторы на 36,0 Ом, 50,0 Ом и 700 Ом?

4. Тостер на 1800 Вт, электрическая сковорода на 1400 Вт и лампа на 75 Вт подключены к одной розетке в цепи 15 А, 120 В.(Три устройства работают параллельно, если подключены к одной розетке.) а) Какой ток потребляет каждое устройство? (b) Перегорит ли эта комбинация предохранитель на 15 А?

5. Фара мощностью 30,0 Вт и стартер мощностью 2,40 кВт обычно подключаются параллельно в систему на 12,0 В. Какую мощность потребляли бы одна фара и стартер при последовательном подключении к батарее 12,0 В? (Не обращайте внимания на любое другое сопротивление в цепи и любое изменение сопротивления в двух устройствах.)

6.(a) Для батареи 48,0 В и резисторов 24,0 Ом и 96,0 Ом найдите для каждого из них ток и мощность при последовательном соединении. (b) Повторите, когда сопротивления включены параллельно.

7. Ссылаясь на пример комбинирования последовательных и параллельных цепей и рисунок 5, вычислите I 3 двумя следующими способами: (a) по известным значениям I и I 2 ; (б) используя закон Ома для R 3 . В обеих частях явно показано, как вы следуете шагам, описанным выше в стратегии решения проблем для последовательных и параллельных резисторов .

Рис. 5. Эти три резистора подключены к источнику напряжения, так что R 2 и R 3 параллельны друг другу, и эта комбинация включена последовательно с R 1 .

8. Ссылаясь на рисунок 5: (a) Вычислите P 3 и обратите внимание на его сравнение с P 3 , найденным в первых двух примерах задач в этом модуле. (b) Найдите полную мощность, отдаваемую источником, и сравните ее с суммой мощностей, рассеиваемых резисторами.

9. См. Рисунок 6 и обсуждение затемнения света при включении тяжелого прибора. (a) Учитывая, что источник напряжения составляет 120 В, сопротивление провода составляет 0,400 Ом, а номинальная мощность лампы составляет 75,0 Вт, какая мощность будет рассеиваться лампой, если при включении двигателя через провода пройдет в общей сложности 15,0 А? Предположите незначительное изменение сопротивления лампы. б) Какая мощность потребляет двигатель?

Рис. 6. Почему гаснет свет, когда включен большой прибор? Ответ заключается в том, что большой ток, потребляемый двигателем прибора, вызывает значительное падение напряжения в проводах и снижает напряжение на свету.

10. Линия электропередачи на 240 кВ, имеющая 5,00 × 10 2 , подвешена к заземленным металлическим опорам с помощью керамических изоляторов, каждый из которых имеет сопротивление 1,00 × 10 9 Ом (рис. 9 (а)). Какое сопротивление на землю у 100 изоляторов? (b) Рассчитайте мощность, рассеиваемую 100 из них. (c) Какая доля мощности, переносимой линией, составляет это? Ясно покажите, как вы следуете шагам, описанным выше в стратегии решения проблем для последовательных и параллельных резисторов .

Рис. 9. Высоковольтная (240 кВ) линия электропередачи 5,00 × 10 2 подвешена к заземленной металлической опоре электропередачи. Ряд керамических изоляторов обеспечивает сопротивление 1,00 × 10 9 Ом каждый.

11. Покажите, что если два резистора R 1 и R 2 объединены, и один из них намного больше другого ( R 1 >> R 2 ): (a ) Их последовательное сопротивление почти равно большему сопротивлению R 1 .(б) Их параллельное сопротивление почти равно меньшему сопротивлению R 2 .

12. Необоснованные результаты Два резистора, один из которых имеет сопротивление 145 Ом, подключены параллельно, чтобы получить общее сопротивление 150 Ом. а) Каково значение второго сопротивления? б) Что неразумного в этом результате? (c) Какие предположения необоснованны или непоследовательны?

13. Необоснованные результаты Два резистора, один из которых имеет сопротивление 900 кОм, соединены последовательно, чтобы получить общее сопротивление 0.500 МОм. а) Каково значение второго сопротивления? б) Что неразумного в этом результате? (c) Какие предположения необоснованны или непоследовательны?

Глоссарий

серия:
последовательность резисторов или других компонентов, включенных в цепь один за другим
резистор:
компонент, обеспечивающий сопротивление току, протекающему через электрическую цепь
сопротивление:
вызывает потерю электроэнергии в цепи
Закон Ома:
соотношение между током, напряжением и сопротивлением в электрической цепи: В = IR
напряжение:
электрическая потенциальная энергия на единицу заряда; электрическое давление, создаваемое источником питания, например аккумулятором
падение напряжения:
потеря электроэнергии при прохождении тока через резистор, провод или другой компонент
ток:
поток заряда через электрическую цепь мимо заданной точки измерения
Закон Джоуля:
взаимосвязь между потенциальной электрической мощностью, напряжением и сопротивлением в электрической цепи, определяемая следующим образом: [latex] {P} _ {e} = \ text {IV} [/ latex]
параллельно:
разводку резисторов или других компонентов в электрической цепи, так что каждый компонент получает одинаковое напряжение от источника питания; часто изображается на диаграмме в виде лестницы, где каждый компонент находится на ступеньке лестницы

Избранные решения проблем и упражнения

1.(а) 2,75 кОм (б) 27,5 Ом

3. (а) 786 Ом (б) 20,3 Ом

5. 29,6 Вт

7. (а) 0,74 А (б) 0,742 А

9. (а) 60,8 Вт (б) 3,18 кВт

11. (a) [латекс] \ begin {array} {} {R} _ {\ text {s}} = {R} _ {1} + {R} _ {2} \\ \ Rightarrow {R} _ {\ text {s}} \ приблизительно {R} _ {1} \ left ({R} _ {1} \ text {>>} {R} _ {2} \ right) \ end {array} \\ [/ латекс]

(b) [латекс] \ frac {1} {{R} _ {p}} = \ frac {1} {{R} _ {1}} + \ frac {1} {{R} _ {2} } = \ frac {{R} _ {1} + {R} _ {2}} {{R} _ {1} {R} _ {2}} \\ [/ latex],

, так что

[латекс] \ begin {array} {} {R} _ {p} = \ frac {{R} _ {1} {R} _ {2}} {{R} _ {1} + {R} _ {2}} \ приблизительно \ frac {{R} _ {1} {R} _ {2}} {{R} _ {1}} = {R} _ {2} \ left ({R} _ {1 } \ text {>>} {R} _ {2} \ right) \ text {.} \ end {array} \\ [/ latex]

13. (a) –400 кОм (b) Сопротивление не может быть отрицательным. (c) Считается, что последовательное сопротивление меньше, чем у одного из резисторов, но должно быть больше, чем у любого из резисторов.

Что такое последовательно-параллельная схема? | Последовательно-параллельные комбинированные схемы

В простых последовательных цепях все компоненты соединены встык, образуя только один путь для прохождения тока по цепи:

В простых параллельных цепях все компоненты подключаются между одними и теми же двумя наборами электрически общих точек, создавая несколько путей для прохождения тока от одного конца батареи к другому:

Правила, касающиеся последовательных и параллельных цепей

Для каждой из этих двух базовых конфигураций схемы у нас есть определенные наборы правил, описывающих отношения напряжения, тока и сопротивления.

Цепи серии :

  • Падения напряжения прибавляются к общему напряжению.
  • Все компоненты имеют одинаковый (равный) ток.
  • Сопротивлений складываются в общую сумму сопротивления.

Параллельные цепи:

  • Все компоненты имеют одинаковое (равное) напряжение.
  • Токи ответвления складываются с равным общим током.
  • Сопротивления уменьшаются до полного сопротивления.
Серия

— Параллельные схемы

Однако, если компоненты схемы соединены последовательно в одних частях и параллельно в других, мы не сможем применить единый набор правил к каждой части этой цепи.Вместо этого нам нужно будет определить, какие части этой цепи являются последовательными, а какие — параллельными, а затем выборочно применять правила последовательного и параллельного подключения по мере необходимости, чтобы определить, что происходит. Возьмем, к примеру, следующую схему:

Эта схема не является ни простой последовательной, ни простой параллельной. Скорее, он содержит элементы обоих. Ток, выходящий из нижней части батареи, разделяется, чтобы пройти через R 3 и R 4 , снова присоединяется, затем снова разделяется, чтобы пройти через R 1 и R 2 , затем снова присоединиться, чтобы вернуться наверх батареи.Существует более одного пути для прохождения тока (не последовательно), но в цепи более двух наборов электрически общих точек (не параллельных).

Поскольку схема представляет собой комбинацию как последовательной, так и параллельной, мы не можем применять правила для напряжения, тока и сопротивления «поперек стола», чтобы начать анализ, как мы могли бы, когда цепи были так или иначе. Например, если бы вышеприведенная схема была простой последовательной, мы могли бы просто сложить R 1 через R 4 , чтобы получить общее сопротивление, вычислить общий ток, а затем вычислить все падения напряжения.Точно так же, если бы вышеуказанная схема была простой параллельной, мы могли бы просто решить для токов ответвления, сложить токи ответвления, чтобы вычислить общий ток, а затем вычислить общее сопротивление из общего напряжения и общего тока. Однако решение этой схемы будет более сложным.

Таблица по-прежнему поможет нам управлять различными значениями для последовательно-параллельных комбинированных цепей, но мы должны быть осторожны, как и где применять разные правила для последовательного и параллельного подключения. Закон Ома, конечно, по-прежнему работает точно так же для определения значений в вертикальном столбце таблицы.

Если мы можем определить, какие части схемы являются последовательными, а какие — параллельными, мы можем анализировать их поэтапно, подходя к каждой части по очереди, используя соответствующие правила для определения отношений напряжения, тока и сопротивления. . Остальная часть этой главы будет посвящена демонстрации техник для этого.

ОБЗОР:

  • Правила последовательных и параллельных цепей должны применяться выборочно к цепям, содержащим оба типа межсоединений.

СВЯЗАННЫЕ РАБОЧИЕ ЛИСТЫ:

Комбинация резисторов — последовательно и параллельно — Учебный материал для IIT JEE


Введение в сочетание резисторов — последовательных и параллельных

Электрический ток — это поток заряженных частиц. Поток зарядов будет постоянным в текущей электроэнергии. Электрический ток течет от более высокого электрического потенциала к более низкому электрическому потенциалу. Для протекания тока требуется замкнутый контур из проводящего материала.Схема состоит из проводов, соединенных встык, и электроны текут в одном направлении.

Схема имеет жилы (провод), выключатель, нагрузку и источник питания. Схема начинается и останавливается в одной и той же точке. Обычно в качестве жил без изоляции используются медные провода. Именно через проводник течет ток. Переключатель используется для размыкания или замыкания цепи. Когда переключатель замкнут, ток течет по цепи, а когда переключатель разомкнут, он размыкает цепь, и ток через нее не течет.Клетка может быть источником энергии. Если мы поместим более одной ячейки, она станет батареей.

Нагрузка, также известная как резистор , использует электрическую энергию и преобразует ее в другую форму энергии. Это может быть лампочка или что-нибудь еще. Если в цепи нет нагрузки, произойдет короткое замыкание.

Схема А

Параметры цепи

Электрический ток измеряется в амперах (амперах) амперметром, который подключается последовательно с другими компонентами в цепи.Ток, I = Q / t, где Q — заряд в кулонах, а t — время в секундах. Андре Мари Ампер обнаружил, что два параллельных провода притягиваются друг к другу, когда электрический ток течет в одном направлении. Кроме того, два параллельных провода отталкиваются друг от друга, когда электрический ток течет в противоположных направлениях. В результате его открытий в этой области единица измерения тока была получена от его имени, которое называется « amp ». Один ампер равен одному кулону заряда в секунду времени.

Напряжение определяется как разность электрических потенциалов между двумя точками в цепи.Единица измерения напряжения — вольт. Устройство происходит от имени Алессандро Вольта. Элементы или батареи обеспечивают необходимое напряжение или разность потенциалов.

Сопротивление препятствует прохождению тока. Это мера способности объекта удерживать поток электронов. Сопротивление будет низким в проводнике и высоким в изоляторе. Измеряется в омах. Единица ома названа в честь ученого Георга Симона Ома, сформулировавшего закон Ома.

Резисторы используются для управления прохождением электрического тока в цепи.Он преобразует электрическую энергию в тепло и свет. Резистор является пассивным компонентом, поскольку он потребляет мощность, но не генерирует ее. Обычно они состоят из металла, углерода или пленки оксида металла. Резисторы используются для ограничения тока и защиты полупроводниковых устройств, таких как светодиоды. Он также используется для ограничения частотной характеристики в цепи фильтра.

Закон Ома

Георг Симон Ом показал взаимосвязь между напряжением, током и сопротивлением и сформулировал закон Ома.Этот закон лежит в основе электричества.

Закон гласит, что V = I R, где напряжение V выражено в вольтах, ток I — в амперах, а сопротивление R — в омах.

Таким образом, I = V / R и R = V / I.

Для большей ясности электричество можно отнести к воде. Таким образом, напряжение в цепи, единицей измерения которой являются вольты, идентично давлению воды, текущей в трубе. Ток в цепи, где единица измерения равен амперам, эквивалентен току воды, протекающей по трубе. Сопротивление в контуре, измеряемое в омах, совпадает с сопротивлением трения и размером трубы, которая ограничивает поток воды.

Закон Ома

Последовательные и параллельные соединения

В основном есть два типа цепей: последовательные и параллельные. И последовательная, и параллельная цепи состоят из более чем одной нагрузки. Резисторы можно подключать последовательно, параллельно или их комбинацию.

В последовательной цепи электронов движутся только по одному пути. Здесь будет тот же ток, который проходит через каждый резистор. Напряжение на резисторе при последовательном включении будет другим.При последовательном соединении, если один резистор сломан или возникает какая-либо неисправность, вся цепь отключается. Последовательные цепи нелегко перегреть. Конструкция последовательной схемы проста по сравнению с параллельной схемой.

Некоторые огни рождественской елки могут быть включены в последовательные цепи. Если погаснет одна лампочка, погаснет вся струна. Предохранитель или автоматические выключатели будут подключены последовательно, чтобы защитить всю проводку от перегрузки по току. Его можно использовать как делитель напряжения. Батарейки в пульте подключены последовательно.

Последовательная цепь

В параллельной цепи электронов проходят через множество ее ветвей. В этом случае напряжение на каждом резисторе в цепи остается неизменным. Здесь ток в цепи делится между каждой ветвью и, наконец, рекомбинирует, когда ветви встречаются в общей точке. Параллельная цепь может быть сформирована разными способами, что означает, что резисторы могут быть расположены в разных формах. Его можно использовать как делитель тока.

В большинстве случаев цепи подключаются параллельно. Это потому, что если один резистор сломан или поврежден, он не отключит всю систему. Но из-за этого эффекта трудно обнаружить отказ, если в цепи что-то пойдет не так, и поэтому в определенные моменты это может быть опасно. Легко подключить или отключить новый резистор или другой компонент, не затрагивая другие элементы в параллельной цепи. Но он использует много проводов и, следовательно, становится сложным. В основном в зданиях и домах мы используем параллельное подключение.

Параллельная цепь

Комбинация резисторов при последовательном включении

Рассмотрим три резистора R 1 , R 2 , R 3 , включенных последовательно. Здесь заряд сначала проходит через 1 рандов, затем переходит в 2 рандов и, наконец, достигает 3 рандов.

Комбинация из трех последовательно соединенных резисторов

По закону Ома разность потенциалов на R 1 = V 1 = I R 1

Разность потенциалов на R 2 = V 2 = I R 2 .

Разность потенциалов на R 3 = V 3 = I R 3 .

Таким образом, разность потенциалов V на этом последовательном соединении резисторов

В = В 1 + В 2 + В 3

= I R 1 + I R 2 + I R 3

= I ( 1 + 2 + 3 )

Таким образом, при последовательном соединении эквивалентное сопротивление R eq = V / I = (R 1 + R 2 + R 3 ).

Для n числа резисторов, соединенных последовательно, эквивалентное сопротивление R eq = R 1 + R 2 + R 3 ………………… R n .

Эквивалентное сопротивление — это полное сопротивление цепи. Это единственное значение сопротивления, которое может заменить количество резисторов в цепи без изменения тока и напряжения в сети. Таким образом, при последовательном соединении общее сопротивление цепи определяется путем сложения сопротивлений каждого отдельного резистора.

Для примера рассмотрим последовательную схему, состоящую из трех резисторов с сопротивлением 5 Ом, 10 Ом, 5 Ом соответственно с батареей 15 В.

Итак, полное сопротивление R = R 1 + R 2 + R 3 = 5 + 10 + 5 = 20 Ом

Мы знаем, что V = I R.

Ток I = V / R = 15/20 = 0,75 А. Хотя ток в последовательной сети одинаков, падение напряжения на каждом резисторе отличается. Каждый резистор с разным сопротивлением обеспечивает разное падение напряжения, и мы можем найти полное напряжение по закону Ома, V = I R.Возьмем предыдущий пример. Упомянутое здесь напряжение составляет 15 В. Мы можем проверить это, рассчитав таким образом. V = I R 1 + I R 2 + I R 3 = 0,75 (5 +10 + 5) = 15 В. Это точное измерение напряжения, которое мы предоставили в этом примере. Обнаружено, что резистор с большим сопротивлением имеет большее падение напряжения.


Комбинация резисторов при параллельном подключении

Рассмотрим три резистора R 1 , R 2 , R 3 , включенных параллельно.Плата делится на три и проходит через 1 рандов, 2 рандов и 3 рандов.

Комбинация из трех параллельно включенных резисторов

Ток I = I 1 + I 2 + I 3.

Разность потенциалов, приложенная к R1 = V = I 1 R 1

Разность потенциалов на R 2 = V = I 2 R 2

Разность потенциалов на R 3 = V = I 3 R 3

Таким образом, I = I 1 + I 2 + I 3

= V / R 1 + V / R 2 + V / R 3

= V (1 / R 1 + 1 / R 2 + 1 / R 3 )

Если эту параллельную комбинацию заменить эквивалентным сопротивлением, R eq

Тогда I = V / R экв

1 / R экв. = 1 / R 1 + 1 / R 2 + 1 / R 3

Таким образом, для n количества резисторов, включенных параллельно, 1 / R eq = 1 / R 1 + 1 / R 2 + 1 / R 3 ……………………. 1 / Р .

Итак, при параллельном соединении полное сопротивление цепи определяется путем сложения обратной величины сопротивления каждого отдельного резистора.

Для примера рассмотрим параллельную схему, состоящую из трех резисторов с сопротивлением 5 Ом, 10 Ом, 5 Ом соответственно с батареей 15 В.

Таким образом, общее сопротивление, 1 / R = 1 / R 1 + 1 / R 2 + 1 / R 3 = 1/5 + 1/10 + 1/5 = 5/10. Итак, R = 2Ω.

При параллельном подключении полное сопротивление или эквивалентное сопротивление всегда будет меньше наименьшего резистора, присутствующего в цепи. Значение эквивалентного сопротивления будет между наименьшим сопротивлением в цепи и наименьшим сопротивлением, деленным на количество резисторов, присутствующих в цепи. В этом примере наименьший резистор имеет сопротивление 5 Ом, а значение общего сопротивления составляет 2 Ом, что явно подтверждает вышеупомянутый факт.

Напряжение на каждом резисторе составляет 15 В.Теперь, чтобы найти ток через каждую ветвь, формула: I = V / R.

I 1 = 15/5 = 3A

I 2 = 15/10 = 1,5 A

I 3 = 15/5 = 3A

Общий ток I = 3 + 1,5 + 3 = 7,5 A


Комбинация последовательных и параллельных резисторов

Рассмотрим схему, в которой R 2 и R 3 подключены параллельно, а R 1 включены последовательно с R 2 и R 3.

Комбинация последовательных и параллельных резисторов


Сначала рассмотрим R 2 и R 3 и, следовательно, 1 / R 23 экв. = 1 / R 2 + 1 / R 3

R 23 экв. = рэндов 2 R 3 / R 2 + R 3

рэндов 123 экв. = рэндов 23 экв + рэндов 1

Таким образом, ток I = V / R 123eq = V / [R 1 + (R 2 R 3 / R 2 + R 3 )]

= V ( рэндов 2 + рэнд 3 ) / рэнд 1 рэнд 2 + рэнд 1 рэнд 3 + р 2 р 3.

Резюме
  • Электрический ток — это поток заряженных частиц в цепи. Схема состоит из проводника, резистора, переключателя и источника питания.

  • Ток, I = Q / t, где Q — заряд в кулонах, а t — время в секундах. Напряжение — это разность электрических потенциалов между двумя точками в цепи. Вольт — это единица измерения напряжения.

  • Сопротивление — это способность управлять потоком электронов в цепи, а единицей измерения сопротивления являются омы.

  • Закон Ома гласит, что V = I R. Таким образом, I = V / R и R = V / I.

  • Есть два типа цепей: последовательные и параллельные.

  • При последовательном соединении ток, протекающий только по одному пути, будет одинаковым при прохождении через каждый резистор.

  • При параллельном подключении напряжение на каждом резисторе в цепи остается неизменным, а ток распределяется между ветвями.


Посмотрите это видео, чтобы получить дополнительную информацию


Другие чтения

Комбинация резисторов — Series и Paralle

резисторов, включенных последовательно и параллельно | Комбинации резисторов

Результаты обучения

  • Рассчитайте общее сопротивление различных комбинаций резисторов, т. Е. Последовательных, параллельных и последовательно-параллельных.
  • Покажите, как резисторы используются в качестве делителей напряжения и тока.
  • Рассчитайте сопротивление и значение мощности для последовательного резистора, понижающего напряжение.

Отдельные резисторы могут быть соединены вместе в последовательном соединении, параллельном соединении или комбинации как последовательного, так и параллельного соединения. Это приводит к более сложной схеме, полное сопротивление которой представляет собой комбинацию отдельных резисторов.

Комбинация резисторов серии

Для подключения резисторов серии , они соединяются встык вместе в одну линию, как показано на рис. 1 .Характеристики последовательно соединенных резисторов можно резюмировать следующим образом:

  • Общее сопротивление цепи ( R T ) увеличивается при последовательном включении дополнительных резисторов и уменьшается при удалении резисторов.
  • Чтобы определить общее сопротивление цепи, просто найдите сумму отдельных сопротивлений нагрузок.
  • В этом примере, если резисторы имеют маркировку R 1 , R 2 и R 3 , то общее сопротивление R T рассчитывается по формуле

Рисунок 1 Резисторы, подключенные последовательно.

ПРИМЕР 1

Проблема: Три резистора, R 1 (4 Ом), R 2 (50 Ом) и R 3 (75 Ом) подключены последовательно, как показано на рис. 2 . Определите значение общего сопротивления комбинированной цепи.

Рисунок 2 Схема для примера 1.

Решение:

Резисторы, соединенные последовательно, используются в качестве делителей напряжения , , как показано на схеме Рисунок 3 .Делители напряжения широко используются в цепях, где один источник напряжения должен обеспечивать несколько разных значений напряжения для разных частей цепи.

Характеристики схемы последовательного делителя напряжения можно резюмировать следующим образом:

  • Через каждый резистор проходит одинаковый ток.
  • Входное напряжение делится пропорционально между последовательно соединенными резисторами.
  • Падение напряжения на резисторе в последовательной цепи прямо пропорционально сопротивлению резистора.
  • Чем выше значение сопротивления, тем больше падение напряжения.

Рисунок 3 Схема делителя напряжения.

Для схемы делителя напряжения падение напряжения на каждом резисторе обычно является фактором, который необходимо определить. Падение напряжения на любом резисторе пропорционально отношению его сопротивления к общему сопротивлению цепи.

Формула для делителя напряжения позволяет рассчитать падение напряжения на любом из последовательно включенных резисторов без необходимости сначала рассчитывать значение тока цепи.Заявлено в виде формулы:

ПРИМЕР 2

Резисторы R 1 (5 кОм), R 2 (3 кОм) и R 3 (2 кОм) ) соединены последовательно, образуя делитель напряжения, как показано на рис. 4 . Если на схему подается входное напряжение 9 вольт, рассчитайте значение падения напряжения на каждом из резисторов, используя формулу делителя напряжения.

Рисунок 4 Цепь для примера 2.

Решение:

ПРИМЕР 3

У вас есть источник на 120 В и вы хотите использовать последовательно понижающий резистор в сочетании с контрольной лампой 6 В при 150 мА для индикации подачи питания ( Рисунок 5 ). Определите значение сопротивления падения серии и требуемую мощность.

Рисунок 5 Схема для примера 3.

Решение:

Параллельная комбинация резисторов

Резисторы соединены параллельно путем соединения бок о бок как показано на рис. 6 .Обратите внимание, что два конца резисторов подключены к одним и тем же двум точкам.

Характеристики параллельно соединенных резисторов можно резюмировать следующим образом:

  • Общее сопротивление (R T ) сформированной цепи на минус , чем сопротивление самого низкого значения сопротивления, присутствующего в любой из ветвей.
  • Каждый резистор обеспечивает отдельный параллельный путь для прохождения тока.
  • Если у вас есть несколько резисторов одинакового номинала, подключенных параллельно, то общее сопротивление легче всего найти, разделив общее значение сопротивления на количество подключенных резисторов.Для трех резисторов по 150 Ом, включенных параллельно, общее сопротивление составляет

Рис. 6 Резисторы , подключенные параллельно.

Чтобы найти полное сопротивление двух неравных значений резисторов, соединенных параллельно (очень распространенное использование), используется формула произведения на сумму. Эта формула:

ПРИМЕР 4

Проблема: Резистор 60 Ом подключается параллельно резистору 40 Ом, как показано на Рисунок 7 .Определите значение общего комбинированного сопротивления двух компонентов, используя формулу «произведение на сумму».

Рисунок 7 Схема для примера 4.

Решение:

Формула произведения на сумму лучше всего работает для двух резисторов, включенных параллельно. Если параллельно подключено более двух резисторов, использование становится труднее и менее практичным. Для более чем двух резисторов разных номиналов, соединенных параллельно, используется общая формула для полного сопротивления параллельной цепи.Эта формула:

ПРИМЕР 5

Проблема: Три резистора, R 1 (120 Ом), R 2 (60 Ом) и R 3 ( 40 Ом) подключены параллельно, как показано на рис. 8 . Определите значение общего сопротивления комбинированной цепи.

Рисунок 8 Цепь для примера 5.

Решение:

Параллельные резистивные цепи можно рассматривать как делители тока , потому что ток разделяется или делится между различными резисторами, как показано на рисунке 9 .

Характеристики схемы параллельного делителя тока можно резюмировать следующим образом:

  • Ток, протекающий через каждый резистор ответвления, обратно пропорционален его значению сопротивления.
  • Чем меньше значение сопротивления, тем больше ток, и наоборот.
  • Резисторы с одинаковым сопротивлением пропускают через них одинаковый ток.
  • Формула, описывающая делитель тока, аналогична формуле для делителя напряжения и может быть выражена следующим образом:

Рисунок 9 Схема делителя тока.

ПРИМЕР 6

Проблема: Резисторы R 1 R 2 и R 3 (2 Ом, 3 Ом и 6 Ом соответственно) подключены параллельно, как показано на Рисунок 10 . Используйте формулу делителя тока, чтобы вычислить значение тока, протекающего через каждый из нагрузочных резисторов, если общий ток, протекающий в цепи, составляет 10 ампер.

Рисунок 10 Цепь для примера 6.

Решение:

ПРИМЕР 7

Проблема: При параллельном подключении дополнительных нагрузок общее сопротивление цепи уменьшается. Для схемы, показанной на рис. 11 , определите полное сопротивление цепи при каждом из следующих рабочих условий:

  1. Переключатели 1 и 2 замкнуты.
  2. Переключатели 1, 2 и 3 замкнуты.

Рисунок 11 Цепь для примера 7.

Решение:

Последовательно-параллельная комбинация резисторов

Комбинированные резистивные цепи, также известные как последовательно-параллельные резистивные цепи , объединяют резисторы последовательно с резисторами, включенными параллельно, как показано на рисунке 12 .

Правила, регулирующие эти цепи, такие же, как и правила, разработанные для последовательных и параллельных цепей. Сначала определяется сопротивление совокупного полного сопротивления параллельной части.Затем общее сопротивление параллельной части добавляется к любому последовательному сопротивлению, чтобы найти общее сопротивление последовательно-параллельной комбинированной цепи.

Рисунок 12 Последовательно-параллельное соединение резисторов.

ПРИМЕР 8

Проблема: Резистор 9 Ом, R 1 , и резистор 60 Ом, R 2 , подключены параллельно друг другу и последовательно с резистор 40 Ом, R 3 , как показано на Рисунок 13 .Определите общее сопротивление этой последовательно-параллельной комбинации резисторов.

Рисунок 13 Цепь для примера 8.

Решение:

ПРИМЕР 9

Проблема: Показания сопротивления можно использовать для проверки цепей на наличие неисправностей. Как определено в предыдущем примере, нормальное полное сопротивление этой последовательно-параллельной схемы цепи , рис. 14, составляет 60 Ом.

  1. Найдите новое значение R T , если резистор R 1 будет поврежден разомкнут , в то время как значения сопротивления R 2 и R 3 останутся равными такой же.
  2. Аналогичным образом найдите новое значение R T , если резистор R 3 будет поврежден закорочен , в то время как значения сопротивления R 1 и R 2 останутся одинаковый.

Рисунок 14 Цепь для примера 9.

Решение:

  1. При разомкнутом отказе R 1 цепь будет состоять из 3 R последовательно с R 2 и всего сопротивление будет:

  1. При коротком замыкании R 3 цепь будет состоять из R 1 параллельно с R 2 , а общее сопротивление будет:

Вопросы для обзора

  1. Рассчитайте общее сопротивление для каждой из следующих цепей резисторов:
    1. Последовательная цепь: R1 = 40 Ом, R2 = 75 Ом
    2. Параллельная цепь: R1 = 200 Ом, R2 = 200 Ом, R3 = 200 Ом
    3. Последовательная цепь: R1 = 2000 Ом, R2 = 6000 Ом, R3 = 2200 Ом
    4. Параллельная цепь: R1 = 14 Ом, R2 = 32 Ом
    5. Последовательная цепь: R1 = 4700 Ом, R2 = 800 Ом, R3 = 200 Ом
    6. Параллельная цепь: R1 = 60 Ом, R2 = 30 Ом, R3 = 15 Ом
  2. 9 0007 Резисторы R 1 , R 2 и R 3 (50 Ом, 30 Ом и 20 Ом соответственно) подключены последовательно через приложенное напряжение 200 В, чтобы сформировать делитель напряжения.Используя формулу делителя напряжения, рассчитайте напряжения E 1 , E 2 и E 3 .
  3. Полный ток на два параллельно соединенных резистора составляет 3 А. Сопротивление R 1 составляет 10 Ом, а сопротивление R 2 составляет 40 Ом. Используя формулу делителя тока, рассчитайте токи I 1 и I 2 .
  4. Резистор 5 Ом, R 1 , и резистор 20 Ом, R 2 , подключены параллельно друг другу и последовательно с резистором 6 Ом, R 3 .Вычислите полное сопротивление этой последовательно-параллельной цепи.
  5. Вам даны три резистора по 100 Ом, которые нужно соединить вместе. Опишите три возможные конфигурации цепей и вычислите их значения общего сопротивления.

Контрольные вопросы — ответы

  1. (a) 115 Ом, (b) 66,7 Ом, (c) 10 200 Ом, (d) 9,74 Ом, (e) 5700 Ом, (f) 8,57 Ом
  2. E 1 = 100 В, E 2 = 60 В, E 3 = 40 В
  3. I 1 = 2.4 A, I 2 = 0,6 A
  4. 10 Ом
  5. Три резистора по 100 Ом, подключенных последовательно, общее сопротивление 300 Ом. Три резистора по 100 Ом, подключенные параллельно, общее сопротивление 33,3 Ом. Два резистора 100 Ом, подключенные параллельно, а затем последовательно соединенные с резистором 100 Ом, общее сопротивление 150 Ом.

4.1 Последовательные и параллельные резисторы

Последовательные резисторы

Когда резисторы включены последовательно? Резисторы включены последовательно всякий раз, когда поток заряда, называемый током, должен проходить через устройства последовательно.Например, если ток течет через человека, держащего отвертку, в землю, тогда R1.R1. на рисунке 4.2 (a) может быть сопротивлением вала отвертки, R2R2 сопротивлением ее ручки, R3R3 сопротивлением тела человека, и R4R4 сопротивление ее обуви.

На рисунке 4.3 показаны резисторы, последовательно подключенные к источнику напряжения. Кажется разумным, что полное сопротивление является суммой отдельных сопротивлений, учитывая, что ток должен проходить через каждый резистор последовательно.Этот факт был бы преимуществом для человека, желающего избежать поражения электрическим током, который мог бы уменьшить ток, надев обувь на резиновой подошве с высоким сопротивлением. Это могло быть недостатком, если бы одно из сопротивлений было неисправным шнуром с высоким сопротивлением к прибору, который уменьшал бы рабочий ток.

Рисунок 4.3 Три резистора, подключенных последовательно к батарее (слева) и эквивалентному одиночному или последовательному сопротивлению (справа).

Чтобы убедиться, что последовательно включенные сопротивления действительно складываются, давайте рассмотрим потерю электроэнергии, называемую падением напряжения, в каждом резисторе на Рисунке 4.3.

Согласно закону Ома падение напряжения V, V на резисторе при протекании через него тока рассчитывается по формуле V = IR, V = IR, где II равно току в амперах (A), а RR — это сопротивление в Ом Ω.Ω. Другой способ представить это: VV — это напряжение, необходимое для протекания тока II через сопротивление RR

.

Таким образом, падение напряжения на R1R1 равно V1 = IR1, V1 = IR1, что на R2R2 равно V2 = IR2, V2 = IR2, а на R3R3 равно V3 = IR3.V3 = IR3. Сумма этих напряжений равна выходному напряжению источника; то есть

4.1 В = V1 + V2 + V3.V = V1 + V2 + V3.

Это уравнение основано на сохранении энергии и сохранении заряда. Электрическая потенциальная энергия может быть описана уравнением PE = qV, PE = qV, где qq — электрический заряд, а VV — напряжение. Таким образом, энергия, подводимая источником, равна qV, qV, а энергия, рассеиваемая резисторами, равна

. 4.2 qV1 + qV2 + qV3. QV1 + qV2 + qV3.

Связи: законы сохранения

Вывод выражений для последовательного и параллельного сопротивления основан на законах сохранения энергии и сохранения заряда, которые гласят, что общий заряд и полная энергия постоянны в любом процессе.Эти два закона непосредственно участвуют во всех электрических явлениях и будут многократно использоваться для объяснения как конкретных эффектов, так и общего поведения электричества.

Эти энергии должны быть равны, потому что в цепи нет другого источника и другого назначения для энергии. Таким образом, qV = qV1 + qV2 + qV3.qV = qV1 + qV2 + qV3. Заряд qq аннулируется, давая V = V1 + V2 + V3, V = V1 + V2 + V3, как указано. (Обратите внимание, что одинаковое количество заряда проходит через батарею и каждый резистор за заданный промежуток времени, поскольку нет емкости для хранения заряда, нет места для утечки заряда и заряд сохраняется.)

Теперь замена значений отдельных напряжений дает

4,3 В = IR1 + IR2 + IR3 = I (R1 + R2 + R3). V = IR1 + IR2 + IR3 = I (R1 + R2 + R3).

Обратите внимание, что для эквивалентного одиночного последовательного сопротивления Rs, Rs мы имеем

Это означает, что полное или эквивалентное последовательное сопротивление RsRs трех резисторов равно Rs = R1 + R2 + R3.Rs = R1 + R2 + R3.

Эта логика действительна в общем для любого количества резисторов, включенных последовательно; таким образом, полное сопротивление RsRs последовательного соединения равно

4.5 Rs = R1 + R2 + R3 + …, Rs = R1 + R2 + R3 + …,

, как предлагается. Поскольку весь ток должен проходить через каждый резистор, он испытывает сопротивление каждого, а последовательно соединенные сопротивления просто складываются.

Пример 4.1 Расчет сопротивления, тока, падения напряжения и рассеиваемой мощности: анализ последовательной цепи

Предположим, что выходное напряжение батареи на рисунке 4.3 составляет 12,0 В и 12,0 В, а сопротивления равны R1 = 1,00 Ом, R1 = 1,00 Ом, R2 = 6,00 Ом, R2 = 6,00 Ом и R3 = 13,0 Ом. R3 = 13,0 Ом. . а) Каково полное сопротивление? (б) Найдите ток.(c) Рассчитайте падение напряжения на каждом резисторе и покажите, что в сумме они равны выходному напряжению источника. (d) Рассчитайте мощность, рассеиваемую каждым резистором. (e) Найдите выходную мощность источника и покажите, что она равна общей мощности, рассеиваемой резисторами.

Стратегия и решение для (а)

Общее сопротивление — это просто сумма отдельных сопротивлений, определяемая уравнением

. 4,6 Rs = R1 + R2 + R3 = 1,00 Ом + 6,00 Ом + 13,0 Ом = 20,0 Ом. Rs = R1 + R2 + R3 = 1.00 Ом + 6,00 Ом + 13,0 Ом = 20,0 Ом.

Стратегия и решение для (b)

Ток определяется по закону Ома, V = IR.V = IR. Ввод значения приложенного напряжения и общего сопротивления дает ток для цепи.

4,7 I = VRs = 12,0 В 20,0 Ом = 0,600 AI = VRs = 12,0 В 20,0 Ом = 0,600 A

Стратегия и решение для (c)

Напряжение — или падение IRIR — на резисторе определяется законом Ома. Ввод тока и значения первого сопротивления дает

. 4.8 V1 = IR1 = (0,600 A) (1,0 Ом) = 0,600 В. V1 = IR1 = (0,600 A) (1,0 Ом) = 0,600 В.

Аналогично

4,9 В2 = IR2 = (0,600 А) (6,0 Ом) = 3,60 В V2 = IR2 = (0,600 А) (6,0 Ом) = 3,60 В

и

4,10 V3 = IR3 = (0,600 A) (13,0 Ом) = 7,80 В. V3 = IR3 = (0,600 A) (13,0 Ом) = 7,80 В.

Обсуждение для (c)

Три капли IRIR в сумме дают 12,0 В, 12,0 В, как и предполагалось.

4,11 V1 + V2 + V3 = (0,600 + 3,60 + 7,80) V = 12,0 VV1 + V2 + V3 = (0,600 + 3,60 + 7,80) V = 12,0 В

Стратегия и решение для (d)

Самый простой способ рассчитать мощность в ваттах (Вт), рассеиваемую резистором в цепи постоянного тока, — это использовать закон Джоуля, P = IV, P = IV, где PP — электрическая мощность.В этом случае через каждый резистор протекает одинаковый полный ток. Подставляя закон Ома V = IRV = IR в закон Джоуля, мы получаем мощность, рассеиваемую первым резистором, как

4,12 P1 = I2R1 = (0,600 A) 2 (1,00 Ом) = 0,360 Вт. P1 = I2R1 = (0,600 A) 2 (1,00 Ом) = 0,360 Вт.

Аналогично

4,13 P2 = I2R2 = (0,600 A) 2 (6,00 Ом) = 2,16 WP2 = I2R2 = (0,600 A) 2 (6,00 Ом) = 2,16 Вт

и

4,14 P3 = I2R3 = (0,600 A) 2 (13,0 Ом) = 4,68 Вт. P3 = I2R3 = (0,600 A) 2 (13,0 Ом) = 4,68 Вт.

Обсуждение для (д)

Мощность также можно рассчитать, используя P = IVP = IV или P = V2R, P = V2R, где VV — это падение напряжения на резисторе (а не полное напряжение источника).Будут получены те же значения.

Стратегия и решение для (e)

Самый простой способ рассчитать выходную мощность источника — использовать P = IV, P = IV, где VV — напряжение источника. Это дает

4,15 P = (0,600 A) (12,0 В) = 7,20 Вт. P = (0,600 A) (12,0 В) = 7,20 Вт.

Обсуждение для (e)

По совпадению обратите внимание, что общая мощность, рассеиваемая резисторами, также составляет 7,20 Вт, что соответствует мощности, выдаваемой источником. То есть

4.16 P1 + P2 + P3 = (0,360 + 2,16 + 4,68) W = 7,20 Вт. P1 + P2 + P3 = (0,360 + 2,16 + 4,68) W = 7,20 Вт.

Мощность — это энергия в единицу времени (ватт), поэтому для сохранения энергии требуется, чтобы выходная мощность источника была равна общей мощности, рассеиваемой резисторами.

Основные характеристики резисторов серии

  1. Последовательные сопротивления складываются Rs = R1 + R2 + R3 + …. Rs = R1 + R2 + R3 + ….
  2. Одинаковый ток протекает последовательно через каждый резистор.
  3. Отдельные последовательно включенные резисторы не получают полное напряжение источника, а скорее делят его.

Разница между последовательными и параллельными цепями

Под последовательностью цепей понимается цепь, имеющая только один путь, по которому протекает ток. В последовательной цепи все компоненты соединены таким образом, что при возникновении неисправности в цепи ток не будет течь через цепь. Ток в последовательной цепи одинаков во всей цепи. С другой стороны, параллельные цепи относятся к схеме с более чем одним путем, по которому протекает ток.В параллельной цепи все компоненты имеют различные ответвления для прохождения тока; таким образом, ток в цепи неодинаков. Прочтите данное руководство, чтобы узнать разницу между последовательными и параллельными цепями.

Что такое последовательная цепь?

Цепь называется последовательной, если ток одинаков во всех компонентах цепи. В последовательных цепях ток имеет только один путь.

В последовательной цепи отношения между током и напряжением прямо противоположны отношениям в параллельной цепи.Ток через каждый последовательный элемент одинаков и равен току источника (Is). Напротив, напряжение на каждом последовательном элементе (V1, V2. V3) изменяется в зависимости от импеданса (в данном примере сопротивления) каждого элемента. Применяется закон Кирхгофа о напряжении (KVL), и напряжение, подаваемое источником (Vs), равно сумме индивидуальных падений напряжения на каждом последовательном элементе.

Преимущество последовательных цепей

  1. Обладает простой и понятной конструкцией.
  2. Не перегревается быстро.
  3. Он имеет более высокое выходное напряжение, поэтому мы можем добавить больше электроприборов.
  4. Он проводит одинаковый ток по всей цепи.

Недостаток последовательной цепи

  1. Если общее количество компонентов в цепи увеличивается, сопротивление цепи увеличивается.
  2. Если отказ происходит в одной точке, вся цепь разрывается.

Теперь рассмотрим пример последовательной цепи, чтобы понять концепцию.

Найдите ток, протекающий через резисторы R1, R2 и R3.

Дано

R1 = 6 Ом

R2 = 6 Ом

R3 = 6 Ом

А V = 36V

Ответ:

Применяя закон Ом в данной цепи, получаем

В = ИК

В = I (R1 + R2 + R3)

Я = В / (R1 + R2 + R3)

I = 36 / (6 + 6 + 6)

I = 36/18

I = 2 А

Что такое параллельная цепь?

Параллельная цепь — это цепь с двумя или более двумя путями прохождения тока.В параллельной цепи все компоненты имеют одинаковое напряжение.

В параллельной цепи напряжение на каждом элементе одинаково и равно напряжению источника (Vs), а ток через каждый элемент (I1, I2, I3) изменяется в соответствии с импедансом (в этом примере сопротивление) каждого элемента. Применяется закон Кирхгофа (KCL) _, и общий ток, протекающий от источника (Is), равен сумме отдельных токов, протекающих через каждый параллельный элемент.

Преимущество параллельных цепей

  1. В параллельной цепи, если какой-либо компонент повреждается, ток не прекращается и продолжает течь через другие компоненты; следовательно, другие компоненты работают эффективно.
  2. В параллельной цепи напряжение на всех компонентах одинаково; поэтому все компоненты работают эффективно.
  3. В параллельной цепи вы можете легко подключить или отключить новый компонент, не влияя на работу другого компонента.

Недостаток параллельной цепи

  1. В параллельной схеме нельзя подать дополнительный источник напряжения.
  2. Параллельная схема требует большого количества проводов для подключения.

Теперь рассмотрим пример параллельной схемы, чтобы понять концепцию.

Найдите полное сопротивление между точками P и Q

Ответ:

Здесь сопротивление 2 Ом, подключенное параллельно с резистором 3 Ом, дает 6/5 Ом. Теперь резистор 6/5 Ом подключен последовательно с сопротивлением 5 и 4 Ом, поэтому общее сопротивление между точками P и Q = 6/5 + 5 + 4 = 10,2 Ом.

Разница между последовательной и параллельной цепями


Последовательная цепь Параллельная цепь
Цепь называется последовательной, если ток одинаков во всех компонентах цепи. Под параллельной схемой понимается цепь с двумя или более двумя путями прохождения тока.
Если отказ происходит в одной точке, вся цепь разрывается. В параллельной цепи, если какой-либо компонент повреждается, ток не прекращается и продолжает течь через другие компоненты; следовательно, другие компоненты работают эффективно.
В последовательной схеме все компоненты расположены в одну линию. В параллельной схеме все компоненты расположены параллельно друг другу.
Если более одного резистора подключены последовательно, напряжение на каждом резисторе не будет одинаковым, хотя ток будет одинаковым по всей цепи. Если резисторы соединены параллельно, напряжение на каждом из резисторов будет одинаковым.
Если V — полное напряжение на всех компонентах в последовательной цепи, оно равно V1 + V2 + V3. Если V — полное напряжение на всех компонентах в параллельной цепи, оно равно V1 = V2 = V3
В последовательной цепи R = R1 + R2 + R3 В параллельной цепи, R = 1 / R1 + 1 / R2 + 1 / R3

Введение в последовательные, параллельные и последовательно-параллельные соединения

Последовательные, параллельные и последовательно-параллельные схемы, их сравнение и применения

Почему параллельное соединение предпочтительнее последовательного?

Сегодня невозможно переоценить использование, применение и важность последовательного и параллельного соединения цепей.Применение последовательного и параллельного подключения цепей можно увидеть в наших домах, школьных залах и в наших уличных фонарях. Одним нажатием кнопки включаются все качели в наших гостиных. некоторые говорят, что у бобов в их домах должны быть разные переключатели.

Что ж, это не волшебство, когда одним переключателем управляет более трех электрических бобов или грузов. Нагрузка — это что угодно, то есть это могут быть приборы, электрические качалки или даже потолочные вентиляторы, которые потребляют электроэнергию при подключении к источнику питания.Электрические бобы, телевизоры, холодильники и т. Д. Можно назвать грузом. Бобы преобразуют электрическую энергию в световую и тепловую форму энергии. Вентиляторы преобразуют электрическую энергию в механическую.

Тип подключения наших потолочных вентиляторов и электрических бобов определит, будут ли они иметь общий выключатель или нет. Последовательное соединение цепи дает нам возможность подключить более двух нагрузок к общему выключателю. Уличные фонари — очень хороший тому пример. Параллельное соединение цепи позволяет нам подключать нагрузки к их индивидуальному переключателю.Подходит как последовательное, так и параллельное соединение, но одно предпочтительнее другого по той или иной причине. Прежде чем мы поговорим о том, почему параллельное соединение предпочтительнее последовательного, давайте вспомним, какие последовательные и параллельные соединения являются первыми.

Последовательная цепь

Последовательная цепь — это цепь, в которой резисторы или нагрузки подключены встык, так что в цепи будет только один путь, по которому протекает электрический ток.Таким образом, когда несколько резисторов соединены последовательно, эффективное сопротивление (общее сопротивление в цепи) получается путем алгебраического сложения отдельных сопротивлений. То есть, если у нас есть резисторы с сопротивлением R1, R2, R3… Rn , соединенные последовательно , то;

R eff = R T = R 1 + R 2 + R 3 +… R n .

При последовательном соединении один и тот же ток течет по всем ветвям цепи, но разное напряжение на нем, что заставляет резисторы иметь разное напряжение на них.На каждом резисторе или нагрузке будет падение напряжения. Приложенное напряжение равно сумме падений напряжения на разных частях цепи. Падение напряжения пропорционально тому, что ток сопротивления одинаков во всей цепи. Когда нагрузки подключаются последовательно, они, как правило, имеют общий выключатель. Такая связь используется в школьных залах, уличных фонарях.

Как подключить фары последовательно?
Использование и применение последовательного соединения

Некоторые люди подключают сигнальные огни в своих домах последовательно, в результате чего у них будет общий выключатель.Проблема с таким типом подключения заключается в том, что при возникновении проблемы с нагрузкой другая подключенная система выйдет из строя. Это тип подключения по схеме «все или ничего». Пока нагрузка не получит энергию до того, как она передаст ее другой, и одна из них не выйдет из строя, будет отключение электроэнергии.

Последовательные соединения схем распространены и широко используются в электрическом оборудовании. Нити трубки в небольших радиоприемниках обычно идут последовательно. Устройства управления током всегда подключаются последовательно с устройством, которое они защищают.Предохранители соединены последовательно с устройством, которое они защищают. Автоматическое отопительное оборудование имеет термостат, электромагнитные катушки и предохранительные выключатели, соединенные последовательно с источником напряжения и т. Д.

Недостатки последовательной цепи
  • Разрыв в проводе отказ или удаление любой отдельной лампы приведет к разрыву цепи и приведению к прекращению работы всех остальных, поскольку в цепи протекает только один единственный путь тока.
  • При добавлении дополнительных ламп в цепь последовательного освещения все они будут иметь меньшую яркость.потому что напряжение распределяется по последовательной цепи. Если мы добавим больше нагрузок в последовательную цепь, падение напряжения возрастет, что не является хорошим признаком для защиты электроприборов. Проводка серии
  • представляет собой проводку типа «ВСЕ или НЕТ», что означает, что все устройства будут работать одновременно или все они отключатся, если произойдет сбой в любом из подключенных устройств в последовательной цепи.
  • Высокое напряжение питания необходимо, если нам нужно добавить дополнительную нагрузку (лампочки, электрические обогреватели, кондиционер и т. Д.) В последовательную цепь.Например, если пять ламп 220 В должны быть подключены последовательно, то напряжение питания должно быть: 5 x 220 В = 1,1 кВ.
  • Общее сопротивление последовательной цепи увеличивается (а ток уменьшается), когда в цепь добавляется дополнительная нагрузка.
  • В соответствии с будущими потребностями, в последовательную цепь тока следует добавлять только эти электроприборы, если они имеют тот же номинальный ток, что и ток, одинаковый в каждой точке последовательной цепи. Однако мы знаем, что электрические приборы и устройства i.е. лампочки, вентилятор, обогреватель, кондиционер и т. д. имеют разный номинальный ток, поэтому их нельзя подключать последовательно для бесперебойной и эффективной работы. Фары, подключенные последовательно
Преимущества последовательного подключения
  • Меньше размер провода кабеля требуется при последовательном подключении.
  • Мы используем для защиты цепи для последовательного подключения предохранителей и автоматических выключателей с другими приборами. Цепи серии
  • нелегко получить накладные расходы из-за высокого сопротивления, когда в цепь добавляется дополнительная нагрузка.
  • Срок службы батареи в последовательной цепи больше, чем в параллельной.
  • Это самый простой способ подключения электропроводки, который позволяет легко обнаружить и устранить неисправность по сравнению с параллельным или последовательно-параллельным подключением.

Параллельная цепь

Резисторы, нагрузки считаются подключенными параллельно, когда конец каждого из резисторов или нагрузок имеет общую точку или соединение, а другие концы также подключены к общей точке или переходу.Такие схемы известны как параллельные схемы.

Лампочки, подключенные параллельно

В отличие от последовательного подключения, при нахождении общего (эффективного) сопротивления в параллельной цепи берется величина, обратная отдельному сопротивлению. Таким образом, когда несколько сопротивлений соединены параллельно, величина, обратная величине эффективного сопротивления, определяется арифметической или алгебраической суммой величины, обратной величине отдельного сопротивления.

1 / R eff или 1 / R T = 1 / R 1 + 1 / R 2 + 1 / R 3 … 1 / R n .

Параллельное соединение цепи имеет одинаковое напряжение, протекающее по всем ветвям цепи. У разных резисторов свои токи.

Использование и применение параллельного соединения

Параллельное соединение цепи очень распространено. Различные лампы и электроприборы в наших домах подключаются параллельно, так что каждая из ламп или бобышек и приборов может работать независимо. Чтобы мы могли управлять отдельными лампами или нагрузками, они должны быть подключены параллельно.

Преимущества параллельной схемы
  • Каждое подключенное электрическое устройство и устройство не зависят от других. Таким образом, включение / выключение устройства не повлияет на другие устройства и их работу.
  • В случае обрыва кабеля или снятия какой-либо лампы все цепи и подключенные нагрузки не разорвутся, другими словами, другие светильники / лампы и электроприборы по-прежнему будут работать без сбоев.
  • Если добавить больше ламп в параллельные цепи освещения, их яркость не будет уменьшена (как это происходит только в цепях последовательного освещения).Потому что напряжение одинаково в каждой точке параллельной цепи. Короче говоря, они получают то же напряжение, что и напряжение источника.
  • Можно добавить дополнительные осветительные приборы и точки нагрузки в параллельных цепях в соответствии с будущими потребностями, если цепь не будет перегружена.
  • Добавление дополнительных устройств и компонентов не приведет к увеличению сопротивления, но уменьшит общее сопротивление цепи, особенно когда используются устройства с высоким номинальным током, такие как кондиционер и электрические обогреватели.
  • параллельная проводка более надежна, безопасна и проста в использовании. Неисправности в параллельных цепях освещения
Недостатки параллельных подключений
  • Кабель и провод большего размера используются в параллельной цепи освещения.
  • При добавлении дополнительной лампочки в параллельную цепь требуется больше тока.
  • Батарея разряжается быстрее при установке постоянного тока.
  • Схема параллельного подключения более сложна по сравнению с последовательным подключением.

Связанное сообщение: Какая лампа светится ярче при последовательном и параллельном подключении и почему?

Последовательно-параллельные соединения и схемы

Схема не является последовательной или параллельной на следующем рисунке, т.е. это последовательно-параллельная схема. Первые три лампы (B 1 , B 2 и B 3 ) подключены параллельно, а переключатели (S 1 , S 2 и S 3 ) подключены последовательно соответственно.B 7 , B 8 , B 9 и B 10 последовательно соединены друг с другом, в то время как они параллельны первым трем лампочкам (B 1 , B 2 и B 3 ) в то время как переключатели (S5 и S6) подключены параллельно к лампе (B 10 ). Кроме того, лампы (B 4 , B 5 и B 6 ) и выключатель (S 7 ) включены последовательно друг с другом, в то время как они параллельны (B 1 , B 2 и B 3 ) и так далее.

Поскольку схема является комбинацией последовательной и параллельной, мы не можем упростить ток, напряжение, сопротивление и мощность с помощью простого закона Ома. Мы должны применить различные теоремы, такие как теоремы Нортона, Тевенина, теоремы о максимальной передаче мощности и т. Д., Или упростим схему в основных последовательных и параллельных схемах, чтобы найти все эти величины.

Наиболее распространенная в настоящее время установка бытовой электропроводки с использованием этого метода электропроводки.

Последовательная параллельная световая цепь и соединение

Сравнение последовательного и параллельного подключения

Ниже в данной таблице показаны основные различия между последовательным и параллельным подключением.

9 2629 Чтобы найти электрическую мощность (P)
S No Последовательная цепь Параллельная цепь
Ток (I) Ток одинаковый в каждой точке последовательной цепи: 1 I 9004 2 = I 3 =…. I n

Ток в последовательной цепи складывается:

I 1 + I 2 + I 3 +…. I n

Напряжение (В) Напряжение складывается в последовательной цепи:

В 1 + В 2 + В 3 +….В n

Напряжения одинаковы в каждой точке параллельной цепи:

В 1 = В 2 = В 3 =…. V n

Сопротивление (R) и найти (R) Сопротивления складываются в последовательной цепи:

R 1 + R 2 + R 3 +… R n = R eff = R T

Сопротивление делится при увеличении нагрузки в цепи.

1 / R T = 1 / R 1 + 1 / R 2 + 1 / R 3 … 1 / R n

или

I = G 1 + G 2 + G 3 +… G n

Чтобы найти ток (I) I = V 1 / R 1 = V 2 / R 2 = V 3 / R 3 = V n / R n I = V 1 / R 1 + V 2 / R 2 + V 3 / R 3 + V n / R n
Для определения напряжения (В) V = I 1 R 1 + I 2 R 2 + I 3 R 3 +… I n R n V = I 1 R 1 = I 2 R 2 = I 3 R 3 =… I n R н.

P = I 2 R 1 + I 2 R 2 +… I 2 R n

или

P = V 1 2 / R 1 + V 2 2 / R 2 +… V n 2/ R n

P = V 2 / R 1 + V 2 / R 2 +… V 2/ R n

или

P = I 1 2 R 1 + I 2 2 R 2 +… I n 2 R n

Правило делителя тока и напряжения В 1 = В T (R 1 / R T ), V 2 = V T (R 2 / R T ) I 1 = I T (G 1 / G T 900 34), I 2 = I T (G 2 / G T )
Пути прохождения электрического тока Только один путь Два или более пути
Яркость лампы Диммер, если добавлено больше ламп (P = V x I) Ярче из-за того же напряжения
При обрыве цепи Вся цепь бесполезна Остальная часть цепь по-прежнему будет работать
Состояние батареи Аккумулятор разряжается медленно (емкость аккумулятора в ампер-часах) Быстрая разрядка аккумулятора (время работы аккумулятора и токи)
Приложения Используется для защиты цепи во время подключение предохранителей и автоматических выключателей последовательно с подключенными приборами Используется в большинстве бытовых электропроводок

Преимущества параллельного соединения по сравнению с последовательным соединением

Последовательное соединение — это соединение всех или отсутствующих схем.Это означает, что если одно из устройств выйдет из строя, все другие устройства также выйдут из строя, поэтому этот тип подключения хорош только тогда, когда мы хотим защитить устройство. Когда плавкий предохранитель сгорает, например, из-за высокого тока, тогда устройство подключается к нему. защищает не будет повреждена, потому что ток больше не будет достигать ее. В то время как последовательное соединение является полным или нулевым, параллельное соединение дает вам возможность дать нагрузкам и приборам их индивидуальный переключатель. Параллельное соединение обеспечивает сопротивление протеканию тока по сравнению с последовательным соединением.

Недостатки последовательной схемы освещения

Резисторы сопротивлением 100 Ом и 150 Ом, подключенные параллельно, будут иметь меньшее влияние на электрический ток по сравнению с резисторами 50 Ом и 40 Ом, подключенными последовательно. В электронных устройствах параллельное соединение имеет первостепенное значение. Все элементы в блоке питания подключены параллельно.

Добавить комментарий

Ваш адрес email не будет опубликован. Обязательные поля помечены *